Download as doc, pdf, or txt
Download as doc, pdf, or txt
You are on page 1of 52

The Professional CPA Review School

Main: 3F C. Villaroman Bldg. 873 P. Campa St. cor Espana, Sampaloc, Manila
 (02) 735 8901 / 735 9031 / 0922 861 0191
email add: crc_ace@yahoo.com
Baguio Davao
Rudel Bldg. V, Lower Mabini cor Diego Silang, Baguio City  3/F GCAM Bldg. Monteverde St. Davao City
(074) 442-1440 / 0922-8499196 (082) 285-8805 / 0925-7272223

INTEGRATED REVIEW MAY 2019 BATCH


FIRST MONTHLY EXAMS FAR

INSTRUCTIONS: Select the correct answer for each of the following questions. Mark only one answer
for each item by SHADING corresponding to the letter of your choice on the answer sheet provided.
STRICTLY NO ERASURES ALLOWED. Use Pencil No. 1 or No. 2 only.
FINANCIAL ACCOUNTING & REPORTING
1. Which statement is INCORRECT?
a. Bank overdrafts are permitted when there are more than 1 existing accounts and at least 1
has sufficient funds
b. Stocks does not qualify as cash equivalent except for redeemable preference shares acquired
3 months before redemption date
c. If you have P1,000,000 cash and needed only P800,000 for a smooth operation of the
company for the whole accounting period, you can invest more than P200,000 cash
d. Deposits in closed banks cannot qualify as cash

2. Which statement is CORRECT in relation to Petty cash fund?


a. In using the imprest fund system, the fund is replenished every disbursement
b. The replenishment is equal to the expense receipts
c. The first step in determining the replenishment check is to do a physical count of the
total cash
d. The fluctuating fund system is the one usually followed in handling petty cash transactions

3. Cash in foreign currency must be translated at


a. The current exchange rate
b. The exchange rate at the time of the preparation of the financial statement
c. The current exchange rate at the reporting date
d. The highest exchange rate for the year

4. The following information was included in the bank reconciliation for Roman Co. for July of 2017:
Checks and charges recorded by bank in July, including a July service charge of P2,800,
P932,600; Service charge made by bank in June and recorded in books in July, P1,200;
Customer’s NSF check returned as a bank charge in July (no entry made in books), P6,000;
Customer’s NSF check returned in June, recorded by the company in July, P15,000; Outstanding
checks in July 31, P300,000; Outstanding checks for June, P255,000; Checks issued in July for
P20,000 recorded by the company as, P2,000; Erroneous bank charge in July, P20,000;
Erroneous bank credit in June corrected in July, P30,000 and Erroneous book receipt in June
corrected in July, P5,000. What is the unadjusted disbursement per book on July 31, 2017?
A. P909,400 C. P918,800
B. P922,000 D. P927,600

5. On February 1, 2018, Henson Company factored receivables with a carrying amount of P300,000 to
Agee Company. Agee Company assesses a finance charge of 3% of the receivables and retains 5% of
the receivables. Relative to this transaction, you are to determine the amount of loss on sale to be
reported in the income statement of Henson Company for February. Assume that Henson
factors the receivables on a with guarantee (recourse) basis. The amount of cash received is
A. P276,000 C. P285,000
B. P291,000 D. P300,000

6. Travel advances should be reported as


a. supplies.
b. cash because they represent the equivalent of money.
c. investments.
d. none of these.
7. Which of the following items should not be included in the Cash caption on the balance sheet?
a. Coins and currency in the cash register
b. Checks from other parties presently in the cash register
c. Amounts on deposit in checking account at the bank
d. Postage stamps on hand

8. From the following information in relation to About Time Company’s BPI checking account
 Bank statement balance, August 31, P1,200,000
 Credit memo adjustments: July, P20,000;August, P25,000
 Debit memo adjustments: July, P18,000;August, P15,000
 Outstanding checks: August 1, P40,000; August 31, P60,000
 Deposits in transit: August 1, P30,000; August 31, P33,000
 Check #12345 for P16,000 was recorded by the book as P1,600
 Bank error in recording a disbursement by About Thyme Company for P8,000 was recorded
against About Time Company’s account
 Deposits for About Time Company’s BPI account for P12,000 was recorded under its BDO
account, and deposits for its BDO account for P16,000 was recorded as deposits to its BPI
account
The unadjusted balance per book, cash in bank – BPI at August 31 is
a. 1,172,600 b. 1,181,400 c. 1,189,400 d. 1,193,400

Item Nos. 9 – 11:


Lipa Corporation started business at the beginning of current year. The entity established an allowance
for doubtful accounts estimated at 5% of credit sales. During the year, the entity wrote off P40,000 of
uncollectible accounts.
Further analysis showed that merchandise purchased amounted to P10,000,000 and ending
merchandise inventory was P2,000,000. Goods were sold at 40% above cost.
The total sales comprised 80% sales on account and 20% cash sales. Total collections from
customers, excluding cash sales, amounted to P6,000,000.
9. What is the cost of goods sold?
a. 8,000,000 c. 7,000,000
b. 6,000,000 d. 5,000,000
10. What is the amount of sales on account?
a. 13,400,000 c. 13,000,000
b. 13,440,000 d. 13,500,000
11. What is the net realizable value of accounts receivable at year-end?
a. 7,400,000 c. 6,768,000
b. 7,000,000 d. 6,632,000

12. Usually, if the petty cash fund is not replenished prior to year-end and no appropriate adjusting
entry is made
a. The petty cash account should be returned to the company cashier
b. Expenses will be overstated and cash will be understated
c. Cash will be overstated and expenses will be understated
d. A complete audit is necessary

13. The results of discontinued operations should be presented a single amount in the
A. Statement of changes in equity
B. Statement of financial position and in the notes thereto
C. Income statement, after tax, separately from income from continuing operations
D. Income statement, before tax, separately from income from continuing operations

14. Expense is recognized


A. When it has been paid for
B. When it is probable that economic benefit can be measured reliably
C. When it is probable that the outflow of economic benefit has occurred
D. When it is probable that an outflow of economic benefit has occurred and it can be
measured reliably
FIRST MONTHLY_FAR_SET B PAGE 3

15. An entity began operations on January 1, 2018. The entity has elected to use cash basis of
accounting for tax purposes and accrual basis for financial reporting. The entity reported sales o P
8,750,000 and P 4,000,000 in the tax returns for 2019 and 2018, respectively. The entity reported
accounts receivable of P 1,500,000 and P 2,500,000 on December 31, 2019 and December 31,
2018 respectively.
During 2019, the entity paid suppliers P 4,900,000 and reported the following balances:
December 31 January 31
Inventory 2,600,000 2,900,000
Accounts payable 750,000 500,000
What amount should be reported as sales in the income statements for 2019?
a. 7,250,000 b. 9,750,000 c. 5,500,000 d. 7,750,000

16. A change in accounting policy made by an entity voluntarily shall be


A. Ignored
B. Applied retrospectively
C. Applied currently and prospectively
D. Treated as a change in accounting estimate

17. Which can be classified as cash and cash equivalents?


a. Travel advances
b. Postage stamps
c. Stock certificate
d. Check drawn by the company, but unreleased

18. You obtained the bank statement, paid checks, and other memoranda relating to Lucy Company’s
bank account for December 2018. In reconciling the bank balance at December 31, 2018, you
observed the following facts:
Balance per bank statement, 12/31/18 P1,465,800
Outstanding checks, 12/31/18 624,750
Receipts of 12/31/18, deposited 1/2/18 95,550
Proceeds of bank loan, 12/15/18, discounted for 90 days at 10% per year,
omitted from records 195,000
Deposit of 12/23/18, omitted from bank statement 53,000
Check 733 of Lucky Co., charged by the bank in error to Lucy Co. 82,100
Proceeds of note receivable of Lucy Co. collected by bank, 12/10/18, not
entered in cash records (Principal, P40,000; Interest, P400; Collection
charge, P100) 40,300
Erroneous debit memo of 12/31/18, to charge company’s account with
settlement of bank loan, paid by check no. 9344 on same date Deposit of 100,000
another client on 12/6/18 credited in error to Lucy Co. 25,000
The cash balance per books of Lucy Company on December 31, 2018 is
A. P1,491,000 C. P1,146,700
B. P961,800 D. P911,400

19. Julia Bakery estimates the allowance for uncollectible accounts at 3% of the ending balance of
accounts receivable. During 2018, Julia’s credit sales and collections were P125,000 and
P131,000, respectively. What was the balance of accounts receivable were written off during 2018
and if the allowance account had a balance of P750 on 12/31/18?
A. P5,820 C. P31,000
B. P31,180 D. P25,000

20. An entity shall disclose on the face of the income statement the allocation of profit or loss between
A. Parent and subsidiary companies
B. Ordinary and extraordinary activities
C. Continuing and discontinued operations
D. Non-controlling (minority) interest and owners of the parent

21. Which of the following is not considered cash for financial reporting purposes?
a. Petty cash funds and change funds
b. Money orders, certified checks, and personal checks
c. Coin, currency, and available funds

SET B
FIRST MONTHLY_FAR_SET B PAGE 4

d. Postdated checks and I.O.U.'s

22. Gi’s Company provided the following information on December 31, 2018:
Cash on hand 600,000
Philippine Bank current account 3,000,000
Manila Bank current account 2,000,000
City Bank current account (bank overdraft) (800,000)
Asia Bank saving account for building acquisition 500,000
Asia Bank time deposit, 90 days 3,000,000
Included among the checks drawn by Celine against the Philippine Bank current account and
recorded in December 2018 are:
 Check written and dated December 21, 2018 and delivered to payee on January 5,2019,
P200,000.
 Check written December 26, 2018, dated January 21, 2019, delivered to payee on
December 27, 2017, P350,000.
What total amount should be reported as cash and cash equivalents on December 31, 2018?
a. 9,150,000 c. 8,600,000
b. 9,650,000 d. 7,200,000

23. On December 31, 2018, Ucan Company reported cash of P3,340,000 with the following details:
Undeposited collections 70,000
Cash in bank – BDO (checking account) 700,000
Cash in nank – PNB (overdraft) (50,000)
Undeposited NSF check received from customer,
dated December 1, 2018 20,000
Undeposited check from a customer, dated January 15, 2018 50,000
Cash in bank – BDO fund for payroll 200,000
Cash in bank – BDO saving deposit 150,000
Cash in bank – BDO money market instrument, 90 days 1,500,000
Cash in foreign bank restricted 200,000
Sinking fund cash 500.000
Total 3,340,000
On December 31, 2018, what total amount should be reported as cash and cash equivalents?
a. 2,000,000 c. 2,020,000
b. 2,620,000 d. 3,300,000

24. Tafushnuh Company had the following account balances on December 31, 2018:
Petty Cash Fund 50,000
Cash in Bank - current account 5,500,000
Cash in bank – sinking fund 3,000,000
Cash on hand 700,000
Cash in bank – restricted account for plant addition
and expected to be disbursed in 2018 2,500,000
Treasury bills 3,000,000
The petty cash fund included unreplenished December 2018 petty cash expensive vouchers
P15,000 and employee IOU P6,000.
The cash on hand included a P150,000 customer check payable to Tafushnuh dated January
15, 2019.
In exchange for a guaranteed line of credit, the entity has agreed to maintain a minimum
balance of P300,000 in the unrestricted current bank account
The sinking fund is set aside to settle a bond payable that is due on June 30, 2018.
What total amount should be reported as cash and cash equivalents on December 31, 2018?
a. 12, 000,000 c. 9,079,000
b. 12,250,000 d. 12,079,000

25. Rorschach Company’s checkbook balance at December 31, 2018 was P180, 000. In addition,
Rorschach held the following items in its safe on that date:
a. Check payable to Rorschach dated January 2,2019 in payment of a sale made in
December 2018, included in December 31 checkbook balance, P65,000.
b. Check payable to Rorschach deposited December 15,but returned by the bank on
December 30 stamped “DAIF”. No entry has yet been made to record the return of this
P20,000 check.

SET B
FIRST MONTHLY_FAR_SET B PAGE 5

c. Check drawn on Rorschach’s account, payable to a vendor, dated and recorded on


December 30 but not yet mailed to payee as of December 31,2018 15,000
What is the amount of cash to be reported on Rorschach’s December 31, 2018 statement of
financial position?
a. 120,000 c. 245,000
b. 230,000 d. 110,000

26. Amiel Company reported the following accounts on December 31, 2018
Cash on Hand 2,000,000
Petty Cash Fund 50,000
HSBC current account 3,000,000
PNB current account no. 1 2,300,000
PNB current account no. 2 (overdraft) (600,000)
BPI current account (overdraft) (150,000)
Ayessa Bank (closed bank) (1,000,000)
BSP Treasury bill – 90 days 2,000,000
BPI time deposit – 60 days 1,800,000
Bond Sinking Fund 2,100,000
 The Cash on Hand included a customer post-dated check of 150,000 and postal money
order of 90,000
 The Petty Cash Fund included unreplenished petty cash vouchers for 8,000 and an
employee check for 7,000 dated January 16, 2019
 The BPI time deposit is set aside for acquisition of Land to be made in Early January
2019.
 The Bond Sinking Fund is set aside for payment of bond payable due December 31,
2018
Compute for the total Cash and Cash Equivalents on December 31, 2018
A. 8,585,000 C. 11,635,000
B. 11,135,000 D. 9,535,000

27. Tim Company reported the following information at the end of the current year:
 Investments securities of 1,700,000. These securities are share investments in entities that
are traded in the Philippine Stock Exchange. As a result, the shares are very actively traded
in the market.
 Investments securities of 3,000,000. These securities are government treasury bills. The
treasury bills have a 10-year term and purchased on December 31 at which they had 2
months to go until they mature.
 Cash of 4,800,000 in the form of coins, currency, saving account and checking account
 Investment securities of 2,200,000. These securities are commercial papers or money
market placements.
 The term of the commercial papers is 9 months and they were purchased on December 31 at
which time they had 3 months to go until they mature.
What total amount should be reported at cash and cash equivalent at the end of current year?
A. 10,000,000 C. 7,800,000
B. 11,700,000 D. 4,800,000

28. Which of the following is considered cash?


a. Certificates of deposit (CDs) c. Money market savings certificates
Money market checking
b. accounts d. Postdated checks

29. A cash equivalent is a short-term, highly liquid investment that is readily convertible into known
amounts of cash and
a. is acceptable as a means to pay current liabilities.
b. has a current market value that is greater than its original cost
c. bears an interest rate that is at least equal to the prime rate of interest at the date of
liquidation.
d. is so near its maturity that it presents insignificant risk of changes in interest rates.

30. Bank overdrafts, if material, should be


a. reported as a deduction from the current asset section.
b. reported as a deduction from cash.
c. netted against cash and a net cash amount reported.
d. reported as a current liability.

SET B
FIRST MONTHLY_FAR_SET B PAGE 6

31. Deposits held as compensating balances


a. usually do not earn interest.
b. if legally restricted and held against short-term credit may be included as cash.
c. if legally restricted and held against long-term credit may be included among current assets.
d. none of these.

32. January 1, 2019, Arc Company sold goods for Matt Company. Matt signed a non-interest bearing
note requiring payment of P60,000 annually every December 31 for seven years, The prevailing
rate of interest for this type of note at the date of issuance was 10%. Information on present value
factors is as follows:
Periods Present Value of 1 at 10% PV of Ordinary annuity of 1 at 10%
6 0.56447 4.355226
7 0.51315 4.86842
What should be the amortized cost of the receivable on December 31, 2019?
A. P321,316 B. P292,105 C. P261,316 D. P227,447

33. Fig Corporation provides an allowance for its doubtful accounts receivable. At December 31, 2018,
the allowance account had a balance of P6,000. Each month Fig Corporation accrues bad debts
expense in an amount equal to 2% of credit sales. Total credit sales during 2019 amounted to
P950,000. During 2019 uncollectible accounts receivable totaling P14,000 were written off against
the allowance account while collection of previously written-off amounted to P3,000. An aging of
accounts receivable at December 31, 2019 indicates that an allowance of P12,000 should be
provided for doubtful accounts as of the date.
Accordingly, bad debts expense previously accrued during 2019 should be increased (decreased)
by
a. 2,000 b. 7,000 c. (2,000) d. (7,000)

34. The category "trade receivables" includes


a. advances to officers and employees.
b. income tax refunds receivable.
c. claims against insurance companies for casualties sustained.
d. none of these.

35. Which of the following is not true?


a. The imprest petty cash system in effect adheres to the rule of disbursement by check.
b. Entries are made to the Petty Cash account only to increase or decrease the size of the
fund or to adjust the balance if not replenished at year-end.
c. The Petty Cash account is debited when the fund is replenished.
d. All of these are not true.

36. BES Corporation’s checkbook balance on December 31, 2016 was P200,000. On the same date,
BES held the following items in its safe:
 A P7,000 check payable to BES, dated January 2, 2017, was not included in the December 31
checkbook balance.
 A P5,000 check payable to BES which was deposited December 19 and included in the
December 31 checkbook balance, was returned by the bank on December 30 marked NSF.
The check was re-deposited on January 2, 2017 and cleared on January 9.
 A P32,000 check payable to a supplier and drawn on BES’ account, was dated and
recorded on December 31, but was not mailed until January 19, 2017
In its December 31, 2016 statement of financial position, how much, should BES report as
cash?
A. 232.000 C. 234,000
B. 227,000 D. 207,000

37. In preparing a two-date bank reconciliation ending December 31, you discovered a customer’s
NSF check of P15,000 was returned by the bank in November and recorded by the Company in
December. What would be the correct treatment in the reconciliation?
a. Bank side, add P15,000 to beginning; and add P15,000 to disbursements
b. Book side, deduct P15,000 from beginning; and deduct P15,000 from disbursements
c. Book side, add P15,000 to disbursements; and deduct P15,000 to ending
d. Bank side, add P15,000 to disbursements; and deduct P15,000 to ending

SET B
FIRST MONTHLY_FAR_SET B PAGE 7

38. The failure to properly record an adjusting entry to accrue an expense will result in an:
a. understatement of expenses and an understatement of liabilities.
b. understatement of expenses and an overstatement of liabilities.
c. understatement of expenses and an overstatement of assets.
d. overstatement of expenses and an understatement of assets.

39. Gump Corporation reported the following information at the current year-end:
 Investment securities of P1,200,000. These securities are share investments in entities that
are traded in the Philippines Stock Exchange. As a result, the shares are very actively
traded in the market.
 Investment securities of P1,800,000. These securities are government treasury bills. The
treasury bills have a 10- year term and purchased on December 31 at which time they had
two months to go until they mature.
 Cash of P3,900,000 in the form of coin, currency, saving account and checking account.
 Investment securities of P1,400,000. These securities are commercial papers. The term of
the papers is nine months and they were purchased on December 31 at which time they had
three months to go until they mature.
What total amount should be reported as cash and cash equivalents at the current year-end?
A. 6,900,000 C. 8,300,000
B. 5,700,000 D. 7,100,000

40. Ilwoo Company’s checkbook balance at December 31, 2017 was 200,000. In addition, Ilwoo held
the following items in its safe on that date:
 Check payable to Ilwoo dated January 2, 2018 in payment of a sale made in December
2017, included in December 31 checkbook balance- P75,000.
 Check payable to Ilwoo deposited December 15, but returned by the bank DAIF- P32,000.
 Check drawn on Ilwoo’s account, payable to a vendor, dated and recorded on December 30
but not yet mailed to payee as of December 31, 2017- P18,000.
What is the correct cash balance of the company?
a. P125,000 c. P160,000
b. P111,000 d. P250,000

41. If material, deposits in foreign bank which are subject to foreign exchange restriction shall be
classified
a. Separately as current asset, with appropriate disclosure
b. Separately as noncurrent asset, with appropriate disclosure
c. Be written off as a loss
d. As part of cash and cash equivalents

42. On December 1, 2016, Now Company established an imprest petty cash fund of P15,000 by
writing a check drawn against its general checking account. On December 30, the petty cash box contained
the following:
 Currency and coins P 6,500
 Receipts for miscellaneous expenses 5,500
 Receipts for office supplies 2,000
 Receipts for transportation 1,800
 Empty envelope with an inscription “P1,200 contribution – finance team
On December 31, Now Company wrote a check to replenish the fund. The entry to record the
replenishment would include a
a. Debit to expenses, P 10,500 b. Credit to cash in bank, P 8,500
Credit to cash in bank, P
c. Credit to cash over, P 800 d. 9,700

43. On December 31, 2016, Bones Company’s cash in bank balance were composed of the following
items:
BDO checking account #CA291M 200,000
BPI checking account #CA219D 120,000
BPI checking account #CA222J (30,000)
AUB checking account #CA771B (40,000)
Further analysis of the checking accounts on January 8, 2016 revealed the following:
 A compensating balance of P40,000 in relation to a loan granted by BDO was legally restricted
as to withdrawal.

SET B
FIRST MONTHLY_FAR_SET B PAGE 8

 A check for P20,000 was recorded against BPI account #CA219D on December 27, 2016
remains on hand until January 3, 2017
 A check for P75,000 was recorded against AUB account #CA771B on December 29, 2016 was
delivered to the supplier on December 30, 2016 was still outstanding as of January 8, 2017
The amount included under cash in Bones Company’s December 31, 2016 statement of financial
position is
a. 250,000 b. 270,000 c. 300,000 d. 305,000

44. The following statements relate to Cash


I. Cash may be offset against a liability only if a legal right of offset exists or the deposit of funds in
a restricted account clearly constitutes the legal discharge of the liability
II. When funds are simply segregated by the company for the purpose of discharging a liability, it is
proper to offset cash against the liability
a. True, true b. True, false c. False, true d. False, false

45. The following statements relate to Cash


I. Credit balances in bank accounts as shown in the books which cannot be offset are to be shown
as current liabilities
II. Credit balances in bank accounts can be offset against demand deposit balances in other
accounts in the same bank, as well as, demand deposit balances in other banks
a. True, true b. True, false c. False, true d. False, false

46. If the cash balance in a company’s bank statement is less than the correct cash balance and
neither the company nor the bank has made any errors, there must be
a. Loan proceeds credited by the bank but not yet recorded by the company
b. Outstanding checks
c. Bank charges not yet recorded by the company
d. Deposits in transit

47. In preparing a two-date bank reconciliation ending December 31, you discovered an erroneous
bank debit of P4,000 in November which was corrected by the bank in December. What would be
the correct treatment in the reconciliation?
a. Book side, deduct P4,000 from beginning; add P4,000 to receipts
b. Bank side, deduct P4,000 from beginning; add P4,000 to receipts
c. Bank side, add P4,000 to beginning; deduct P4,000 to receipts
d. Book side, add P4,000 to beginning; deduct P4,000 to receipts

48. Which of the following does not require an adjustment in the depositor’s books?
a. Collection from a customer of P200,000 was deposited, but recorded by the depositor as
P20,000
b. Deposit of another company erroneously credited to the account of our enterprise
c. Check in payment of accounts payable amounting to P40,000 was recorded by the depositor as
P4,000
d. Proceeds of a loan from bank credited to our enterprises’ account by the depository bank

49. Mould Corp. sells to wholesalers on terms of 2/15, n/30. Mould has no cash sales but based on
historical experience 50% of Mould’s customers take advantage of the discount. Mould uses the
gross method of recording sales and trade receivables. An analysis of Mould’s trade receivables
balances at December 31, 2016 revealed the following:
Age Amount Collectibility
0-15 days P300,000 100%
16-30 days 180,000 95%
31-60 days 15,000 90%
Over 60 days 7,500 P1,500
P502,500
In its December 31, 2016 statement of financial position, what amount should Mould report for
provision for discounts?
A. P3,000 B. P4,860 C. P5,025 D. P6,000

SET B
FIRST MONTHLY_FAR_SET B PAGE 9

50. Wellington Corp has an outstanding accounts receivable totaling P6.5 million as of December 31
and sales on credit during the year of P24 million. There is also a credit balance of P12,000 in the
allowance for doubtful accounts. If the company estimates that 8% of its outstanding receivables
will be uncollectible, what will be the amount of bad debt expense recognized for the year?
A. P508,000 B. P520,000 C. P532,000 P1,920,000

51. Sun Inc assigns P2,000,000 of its accounts receivables as collateral for a P1 million 8% loan with
a bank. Sun Inc. also pays a finance fee of 1% on the transaction upfront. What would be recorded
as a gain (loss) on the transfer of receivables?
Non
A. e C. Loss of P160,000
B. Loss of P20,000 D. Loss of P180,000

52. In financial reporting, the term “probable” means that the probability that an event will occur is
A. More than the probability that the event will not occur
B. Less than the probability that the event will not occur
C. Same as the probability that the event will not occur
D. Highly uncertain

53. MIGHTY Company had the following bank reconciliation at March 31, 2019:
Balance per bank statement, 3/31/2019 P37,200
Add: Deposit in transit 10,300
47,500
Less: Outstanding checks 12,600
Balance per books, 3/31/19 P34,900

Data per bank for the month of April 2019 follows:


Deposits P46,700
Disbursements 49,700

All reconciling items at March 31, 2019 cleared the bank in April. Outstanding checks at April 30,
2019 totalled P6,000. There were no deposits in transit at April 30, 2019. What is the cash
balance per books at April 30, 2019?
A. P28,200 C. P31,900
B. P34,200 D. P38,500

54. Sexy Sadie Company had the following items listed in its trial balance at 12/31/18:
Currency and coins P650
Balance in checking account 2,600
Customer checks waiting to be deposited 1,200
Treasury bills, purchased on 11/2/18, mature on 4/30/19 3,000
Marketable equity securities 10,200
Commercial paper, purchased on 11/2/18, mature on 1/30/19 5,000
What amount will Sexy Sadie include in its year-end statement of financial position as cash and
cash equivalents?
A. P9,450 C. P7,450
B. P12,450 D. P19,650

55. It is the process that involves the simultaneous recognition of revenue and expenses that result
directly from the same transactions or events
Matching of cost with
A. revenue C. Matching of revenue of cost
B. Immediate recognition D. Systematic and rational allocation

56. A change in accounting policy as required by an accounting standard shall be


A. Ignored
B. Applied retrospectively
C. Applied currently and prospectively
D. Applied in accordance with the transitional provisions of the related standard

57. Which of the following is an example of an accrued expense?


a. Office supplies purchased at the beginning of the year and debited to an expense account.
b. Property taxes incurred during the year, to be paid in the first quarter of the
subsequent year.

SET B
FIRST MONTHLY_FAR_SET B PAGE 10

c. Depreciation expense.
d. Rent earned during the period, to be received at the end of the year.

Item Nos. 58 – 62:


JJRL Company reported the following balances on December 31, 2018.
Cash on hand P 630,000
Petty cash fund 12,500
BPI, current account 1,200,000
PNB, current account 1 1,380,000
PNB, current account 2 (80,000)
3-month T-Bill 160,000
Worldwide Bank, opened Dec. 1, 2018 $ 15,000
Limitless Bank (Closed) 200,000
Additional info:
• Cash on hand includes check of P130,000 from customer dated 2/11/18 and postal money orders
amounting to P80,000.
• Petty cash fund consists of currency and coins of P6,000, IOU of employee of P3,000, a sealed
envelope marked “P2,500 party collections”, and a company check of P1,200 addressed to the petty
cashier.
• Included in the BPI current account is a check of P70,000 drawn by the company dated 1/6/19 and
delivered 12/28/18. Also included is a P150,000 compensating balance required at all times to insure
future credit availability.
• The exchange rates are as follows: P48, Dec. 1, 2018; P50, Dec. 28, 2018; P53, Dec.31, 2018; P56,
Jan. 8, 2019 (when the statement was prepared).
Determine the balances of the following:
58. Cash on hand
a. 500,000 c. 550,000
b. 630,000 d. 450,000
59. Petty cash fund
a. 12,700 c. 6,200
b. 12,500 d. 6,000
60. BPI current account
a. 1,200,000 c. 1,050,000
b. 1,270,000 d. 1,120,000
61. Worldwide Bank
a. 720,000 c. 795,000
b. 750,000 d. 840,000
62. Cash and Cash Equivalents
a. 4,000,000 c. 4,281,000
b. 3,985,000 d. 4,031,000

63. A Cash Over and Short account


a. is not generally accepted.
b. is debited when the petty cash fund proves out over.
c. is debited when the petty cash fund proves out short.
d. is a contra account to Cash.

64. The journal entries for a bank reconciliation


a. are taken from the "balance per bank" section only.
b. may include a debit to Office Expense for bank service charges.
c. may include a credit to Accounts Receivable for an NSF check.
d. may include a debit to Accounts Payable for an NSF check.

65. When preparing a bank reconciliation, bank credits are


a. added to the bank statement balance.
b. deducted from the bank statement balance.
c. added to the balance per books.
d. deducted from the balance per books.

END OF EXAMINATION
SET B
The Professional CPA Review School
Main: 3F C. Villaroman Bldg. 873 P. Campa St. cor Espana, Sampaloc, Manila
(02) 735 8901 / 735 9031 / 0922 861 0191
email add: crc_ace@yahoo.com
Baguio Davao
Rudel Bldg. V, Lower Mabini cor Diego Silang, Baguio City  3/F GCAM Bldg. Monteverde St. Davao City
(074) 442-1440 / 0922-8499196 (082) 285-8805 / 0925-7272223

INTEGRATED REVIEW FINANCIAL ACCOUNTING & REPORTING

INSTRUCTIONS: Select the correct answer for each of the following questions. Mark only one answer
for each item by Shading the corresponding letter of your choice on the answer sheet provided.
STRICTLY NO ERASURES ALLOWED. Use Pencil No. 2 only.

PROBLEM 1
On January 1, 2017, an entity granted to an employee the right to choose either shares
or cash payment. The choices are as follows:
 Share alternative – equal to 25,000 shares with par value of P30.
 Cash alternative – cash payment equal to the market value of 20,000 shares
The grant is conditional upon the completion of three years of service. On grant date,
on January 1, 2017, the share price is P51.
The share prices for the three-year vesting period are P54 on December 31, 2017, P66
on December 31, 2018 and P65 on December 31, 2019.
After taking into account the effect of vesting restrictions, the entity has estimated
that the fair value of the share
alternative is
P48.
What is the compensation expense for
1. 2017?
a c 420,00 540,00
. 360,000 b. 300,000 . 0 d. 0
What is the compensation expense for
2. 2018?
580,0 d 460,0
a. 880,000 b. 520,000 c. 00 . 00
What is the compensation expense for
3. 2019?
480,0 420,00 600,00 650,0
a. 00 b. 0 c. 0 d. 00
Wh premium if the employee has chosen the
4. at is share share alternative?
730,0 550,00 370,00
a. 00 b. 0 c. 0 d. 0
Wh is the premium if the employee has chosen the cash
5. at share alternative?
180,0 d
a. 875,000 b. 900,000 c. 00 . 0

PROBLEM 2
On January 1, 2017, an entity purchased an equipment for the cash price of P5,000,000.
The supplier can choose
how the purchase to be settled. The choices are P50,000 shares with par value of P50 in
one year’s time, or a cash
payment equal to be the market value of P40,000 shares on December 31, 2017. At
grant date on January 1, 2017, the market price of each share is P110 and on the date
of settlement on December 31, 2017, the market price of each share is P130.
What is the initial cost of the
6. equipment?
5,000, b 2,500,00 4,400,00 5,500,0
a. 000 . 0 c. 0 d. 00
Wh is the component arising from the purchase of equipment with share
7. at equity and cash alternative?
a 400,00 600,00 d
. 500,000 b. 0 c. 0 . 0
5. What is the interest expense to be recognized on December 31, 2017 if the
supplier has chosen the cash alternative?
a 800,0
. 600,000 b. 400,000 c. 00 d. 0
9. What is the share premium on December 31, 2017 if the supplier
has chosen the share alternative?
5,0000,0 P4,400, P4,000,0
a. 00 b. P2,500,000 c. 000 d. 00

PROBLEM 3
On January 1, 2017, an entity offered management share appreciation rights equal to
50,000 shares with a predetermined price of P100. The service period is 3 years and the
exercise date is January 1, 2020. The quoted prices per share are P124 on December
31, 2017, P151 on December 31, 2018 and P151 on December 31, 2019.
10. What amount should be charged to compensation expense for
2019?
d.
a. 2,550,000 b. 1,300,000 c. 850,000 0

6. What amount should be recognized as gain on reversal of share appreciation rights


in 2019 if the market ‘price dropped to P120 on December 31, 2019?
a. 1,000,000 b. 1,700,000 c. 700,000 d. 0
7. Lehman Corporation purchased a machine on January 2, 2013, for P2,000,000. The
machine has an estimated 5-year life with no residual value. The straight-line method
of depreciation is being used for financial statement
purposes and the following accelerated depreciation amounts will be deducted for tax purposes:
201
3 P400,000 2016 P230,000
201
4 640,000 2017 230,000
201
5 384,000 2018 116,000
Assuming an income tax rate of 30% for all years, the net deferred tax liability that should be reflected on Lehman’s
statement of financial position at December 31, 2014, should be
Deferred Tax
Liability
Curre Noncurr
nt ent
P P72,0
A. 0 00
B. P4,800 P67,200
C. P67,200 P4,800
D. P72,000 P0

9. When a change in the tax rate is enacted into law, its effect on existing deferred
income tax accounts should be
 Handled retroactively in accordance with the guidance related to changes in
accounting standards.
 Considered, but it should only be recorded in the accounts if it reduces a
deferred tax liability or increases a deferred tax asset.
 Reported as an adjustment to tax expense in the period of change.
 Applied to all temporary or permanent differences that arise prior to the date
of the enactment of the tax rate change, but not subsequent to the date of the
change.

10.Man Company purchased 10% of Kind Corporation’s 200,000 outstanding shares of


ordinary shares on January 2, 2018 for P2,500,000. On January 2, 2018, Man
Company purchased another 40,000 shares of Kind for P6,000,000. There was no
goodwill as a result of either acquisition Kind reported earnings of P6,000,000 and
P7,000,000 for the year ended December 31, 2018 and December 31, 2019,
respectively. No dividends were declared in years 2018 and 2019, respectively by
Kind Company.
What amount of income from investment should Man Company report in its
statement of comprehensive income related to its investment for the year ended
December 31, 2019?
P600,00
A. None B. 0
P2,100,
C. P1,400,000 D. 000

10.On May 1, 2016, Golden Company purchased a short-term P4,000,00 face value 9%
debt instruments for P3,720,000 excluding the accrued interest and classified it as a
investment to profit or loss which is based on the business model of the entity to buy
and sell portfolio of securities and to make profit for shorter movements in the
market rate of interest. Golden Company incurred and paid P20,000 transaction cost
related to the acquisition of the instrument. The debt instruments mature on January
1, 2019, and pay interest semi-annually on January 1 and July 1. On December 31,
the fair market value of the instruments is P3,880,000. On February
2, 2017, Graham Company sold the debt security for P3,960,000.
What amount should Golden Company report for short-term debt securities on
December 31, 2016?
P3,720,
A. P3,600,000 B. 000
P3,880, P3,960,
C. 000 D. 000
15. On January 2, 2018, Marco Company purchased 200,000 shares (20%) of Polo
Company’s ordinary share for
P4,500,000. During 2018, Polo reported the following in its statement of
comprehensive income a P4,000,00 net income and a P500,000 unrealized gain from
its investment in available for sale. Polo Company paid cash
dividends of P3,000,000 on December 31, 2018. On January 1, 2019, Marco
Company sold 50,000 shares of
Polo Company at the current market value of Polo’s shares at P32 per share.
What amount of gain should Marco Company recognize from the sale of 50,000
shares?
P425,
A. P400,000 B. 000
P500,0
C. P450,000 D. 00

16. Which of the following items is an example of investment property?


← Property that is leased to another entity under a finance lease
← Property that is being constructed or developed on behalf of third parties
← Property that is being constructed or developed for future use as
investment property
← Property held for short-term sale in the ordinary course of business

17.On January 2, 2018, Star Company originates a 10-year 7% P4,000,000. The loan
carries an annual interest rate of 7% and is repayable at par at the end of year 10
(December 31, 2027). Star Company charges a 1.25% (P50,000) non-refundable loan
origination costs. The contract specifies that the borrower has an option to pre-pay
the instrument at approximately equal to instrument’s amortized cost at each
exercise date, and that no penalty will be charged for pre-payment. But at the
inception of the contract, Star Company expects the
borrower not to prepay, the amortization period is equal to the instrument’s full term
and for that reason the effective yield rate is determined at 6.823%.
What is the amortized cost of the instrument on December 31, 2019?
P4,046,
A. P4,050,000 B. 331
P4,042, P4,038,
C. 413 D. 288

18.The Minor Company leased a freehold building for 20 years, the useful life of the
building, with effect from 1 January 2017. At that date the fair value of the leasehold
interest was P7.5 million of which P6.0 million was attributable to the building.
Annual rentals of P800,000 are payable in advance on 1 January.
How much should Minor recognize as an operating lease expense in the year
ended 31 December 2017, according to IAS17 Leases?
P160,
A. Nil B. 000
P800,0
C. P640,000 D. 00

2 A herd of 5 four year old animals was held on 1 January 2017. On 1 July 2017 a 4 ½
year old animal was purchased. The fair values less estimated point of sale costs
were as follows: 4 year old animal at 1 January 2017 P15,000; 4 ½ year old animal at
1 July 2017 P15,900; 5 year old animal at 31 December 2017 P17,250. What amount
should the company recognize in its December 31, 2017 statement of comprehensive
income related to
the animals as a result of the change in their
fair market value?
P12,6
A. P10,000 B. 00
P28,5
C. P15,900 D. 00

19.Island Company owes P2,000,000 plus P180,000 of accrued interest to First State
Bank. The debt is a 10-year, 10% note. During 2017, Island’s business deteriorated
due to a faltering regional economy. On December 31,
2019, First State Bank agrees to accept an old machine and cancel the entire debt.
The machine has a cost of P3,900,000, accumulated depreciation of P2,210,000, and
a fair market value of P1,900,000.
How much should Island Company recognize as a finance income in its profit or loss
as a result of the financial
liability’s derecognition?
P280,
A. P210,000 B. 000
P490,0
C. P310,000 D. 00

20.Silver Company purchased a plot of land with a building at a cost of P10,000,000 in


2015. The land portion accounted for P2,000,000 of the purchase price. The building
is depreciated on a straight-line basis over 50 years, charging a full year’s
depreciation in the year of acquisition and none in the year of disposal. On January 2,
2017, the land and building was revalued upward. An independent professional
valuer placed a valuation of P18,000,000 on the existing use basis, of which
P4,000,000 was attributable to the land portion. The surplus was incorporated in the
accounts.
What is the total amount of revaluation surplus should the company recognize on
January 2, 2017?
P4,000,
A. P2,000,000 B. 000
P8,320,
C. P6,320,000 D. 000

22. Derby Company, a public limited company, has granted share options to its
employees with a fair value of
P12,000,000. The options vest in three years’ time. The company uses the fair value
model to estimate the fair value of the options, the number of employees that will
vest and the reversion of estimates such as the following:
A. Grant date – January 1, 2016, estimate of employees leaving the company
during the vesting period
– 5%
B. Revision of estimate – January 1, 2017 – estimate of employees leaving the
company during the vesting period – 6%
C. Actual number of employees leaving the company – December 31, 2018 –
5%
What would be the amount of expense changed in the income statement for the
year ended December 31, 2018?
P3,800,
A. P3,760,000 B. 000
P3,880, P4,00,0
C. 000 D. 00

d. Salvation Corporation had two (2) issues of securities outstanding – ordinary share
and an 8% convertible bond issue with a face amount of P16,000,000. Interest
payment dates of the bond issue are June 30 and December 31. The conversion
clause in the bond indenture entitles the bondholders to receive forty (40) shares of
P20 par value ordinary share in exchange for each 1,000 bond. On June 30, 2017, the
holders of P2,400,000 face value bonds exercised the conversion privilege. The
equity component of the convertible debt at the time of issue is P950,000. The
market price of the bonds in that date was P1,100 and the market price of the
ordinary share was P35. The total unamortized bond discount at the date of
conversion was P1,000,000.
In applying the book value method, what amount should Salvation credit to the
“Share Premium in Excess of Par” account as a result of this conversion?
A. P160,000 B. P330,000
P472, P1,440,0
C. 500 D. 000

2 Jason Company has taken out a foreign loan of $100,00 that is recorded at
P4,400,00. At the reporting date, the carrying value of the loan is P4,000,000. The
unrealized gain of P400,000 is included in profit or loss, but will be taxable when the
gain is realized on the repayment of the loan.
If the current and future tax rates are 34% and 35% respectively, what amount of
deferred tax asset should the company recognize?
Non P136,0
A. e B. 00
P276,0
C. P140,000 D. 00

23. Dividends in the form of noncash assets are measured at


a Fair value of the assets distributed
b Carrying amount of the assets distributed
c Either the carrying amount or fair value of the assets distributed
d Neither the carrying amount nor fair value of the assets distributed

24.Mountain Company sells a new product. During a move to a new location, the
inventory records for the
product were misplaced. The entity has been able to gather some information
from the purchases and sales records. The July purchases are as follows:
Total
Quantity Unit Cost Cost
July 5 10,000 65 650,000
9 12,000 63 756,000
12 15,000 60 900,000
14,0 868,00
25 00 62 0
51,0 3,174,0
00 00
On July 31, 15,000 units were on hand. The sales for July amount to P6,000,000 or
60,000 units at P100 per unit. The entity has always used a periodic FIFO inventory
costing system. Gross profit on sales for July was P2,400,000. What is the cost of
inventory on July 1?
1,354, 2,826,0
A. 000 C. 00
2,400,0 426,00
B. 00 D. 0

 On July 1, 2013, Jayson Company purchased as a long-term investment P1,000,000 of


Channing Company’s 8% bonds for P946,000, including accrued interest of P40,000.
The bonds were purchased to yield 10% interest. The bonds mature on January 1,
2019, and pay interest annually on January 1. Jayson Company used the effective
interest method of amortization.
On December 31, 2013, what carrying amount of the investment in bonds?
911,3 953,3
A. 00 C. 00
916,6 960,6
B. 00 D. 00

26.The unadjusted physical inventory of Lindsay Company at December 31,


2015 was P 3,000,000. Other information follows:
a. Goods were received and recorded on January 2, 2016 with cost of P 180,000.
Information revealed that the term of the shipment is FOB shipping point and
these goods were shipped on December 29, 2015.
b. Merchandise in the warehouse costing P 240,000 was billed to the customer FOB
shipping point on December 29, 2015. These were excluded from inventory but
these were shipped on January 3, 2016.
How much should Lindsay report as inventory in its December 31, 2015 statement of
financial position?
a. P 3,000,000 b. P 3,180,000 c. P 3,240,000 d. P 3,420,000

30 On December 2, 2014, Holland company purchased goods with a cash Some of the
. price 200,000. costs
incurred in connection with the acquisition of goods were as follows: import duties,
20,000; transportation
costs, 10,000 and handling costs 5,000. These goods were received on December 31,
2014. In Holland’s
December 31, 2014 balance sheet, at what amount should these goods be included in
inventory?
100,0
a. 104,000 b. 107,000 c. 109,000 d. 00
31 An entity reported the following equity at the beginning of the
. current year:
Share capital, P10
par 5,000,000
Share premium 2,000,000
Retained earnings 1,500,000
During the current year, the entity had the following share transactions:
d. Acquired 20,000 treasury shares for P1,000,000
e. Sold 15,000 treasury shares at P60 per share.
f. Sold the remaining treasury shares at P45 per shares.
What amount should be reported as share premium at year –
end?
d.
a. 2,125,000 b. 2,150,000 c. 2,000,000 1,975,000
 During 2017, an entity decided to change from the FIFO method of inventory valuation to the weighted
average method. Inventory balances under each method were:
Weighted
FIFO Average
December 31, 2014 4,500,000 5,400,000
December 31, 2015 7,800,000 7,100,000
December 31, 2016 8,300,000 7,800,000
Assuming income tax rate of 30% what amount should be reported as the effect
of this accounting change in the statement of retained earnings for 2017?
350,000 490,000
a. decrease c. decrease
350,000 490,000
b. increase d. increase

28.Star Company has recently launched a model of consumer car. Its cars are sold with
a three-year warranty for manufacturing defects. Past experience of similar models
indicates that about 10% of the cars sold are with some defects, of which 4% are
minor defects, 3% care normal defects and 3% are major defects. For the year
ended December 31, 2015, the company sold P10,000 units of the new model. The following information relates to
the estimate of costs of defects associated with the new model:
Cost of repair Probabilit Minor Normal Major
unit y defects defects defects
High 30% P1,500 P4,000 P7,000
Medium 60% 1,200 3,000 5,000
Low 10% 1,000 1,500 2,000
What amount of provision should the company recognized for the year ended
December 31, 2015.
P3,043,
a. None b. P1,445,000 c. P1,590,000 d. 000

34. Presented below is the December 31 trial balance of Corinthians Company.


Corinthians
Company
Trial Balance
December 31,
2015
Deb Cred
it it
P14,80
Cash 0
Accounts Receivable 33,600
Allow. For Doubtful Accounts P2,160
Inventory, January 1 62,400
Furniture and Equipment 67,200
Accumulated Depreciation,
January 1 26,880
4,08
Prepaid Insurance 0
Notes Payable 22,400
Owner, Capital 72,000
Sales 480,000
Purchases 320,000
Sales Salaries Expense 40,000
5,36
Advertising Expense 0
Administrative Salaries
Expense 52,000
4,0
Office Expense 00
P603,4 P603,4
40 40

31.Adjust the Allowance for Doubtful Accounts to 8 percent of the accounts


receivable.
32.Furniture and equipment is depreciated at 20 percent per year.
33.Insurance expired during the year, P2,040.
34.Interest accrued on notes payable, P2,688.
35.Sales salaries incurred but not paid, P1,920.
36.Advertising paid in advance, P560.
37.Office supplies on hand, P1,200, charged to Office Expense when
purchased.
38.Inventory on December 31, P64,000.
Disregarding income taxes, the
adjusted profit is
P41,3
A. P39,224 B. 84
P44,0
C. P41,912 D. 72
35. The Retained Earnings account of Lester Corp. for the year 2015 consists of the following items:
Debit Credit
P112,5
Balance, January 1, 2015 00
P6,0
Write-off of organization costs 00
Excess of issuing price of share capital
over par value 24,000
2,50
Loss on the sale of equipment 0
10,5
Correction of error for prior year 00
Gain on sale of treasury shares 3,500
60,0
Cash and share dividends 00
Net income for the year 58,500
119,50
Balance, December 31, 2015 0
P198,0 P198,5
00 00
The correct balance of retained earnings on December 31,
2015 is
B
A. P119,500 . P100,500
D P92,0
C. P94,500 . 00

33.The following account balances relating to property, plant and equipment of


Paperback Company appear on the books on December 31, 2014:
P6,000,
Land 000
60,000,
Building 000
24,000,
Accumulated depreciation 000
Plant, property and equipment have been carried at cost since their acquisition. The
land was acquired 15 years ago while the building’s construction was completed on
January 1, 2005. The straight line method for depreciation is used. On January 1,
2015, the company revalued property, plant and equipment. On the same date,
contracted professional appraisers submitted the following:
Fair
value
P8,000,0
Land 00
Building 48,000,000
What is the revaluation surplus on
December 31, 2015?
P13,200 P15,000,0
A. ,000 B. 00
P14,000,0
C. P13,800,000 D. 00

34.Which of the following is an implication of the going concern assumption?


← The historical cost principle is credible.
← Depreciation and amortization policies are justifiable and appropriate.
← The current-noncurrent classification of assets and liabilities is justifiable and
significant.
← All of these imply the going concern assumption.

35. The economic entity assumption


← Is inapplicable to unincorporated businesses
← Recognizes the legal aspects of business organizations
← Requires periodic income measurement
← Is applicable to all forms of business organizations

36.The inventory on hand at December 31, 2012 for Arrow Company valued at cost of
P947,800. The following items were not included in this inventory amount:
← Purchased goods in transit, shipped FOB destination invoice price P32,000 which
included freight charges of P1,600.
← Goods held on consignment by Arrow Company at a sales price of P28,000,
including sales commission of 20% of the sales price.
← Goods sold to Suits Company, under terms FOB destinations, invoiced for P18,500
which includes P1,000 freight charges to deliver the goods. Goods are in transit.
← Purchased goods in transit, terms FOB shipping point, invoice price P48,000,
freight cost P3,000.
← Goods out on consignment to GG Company, sales price P36,400, shipping cost of
P2,000

Assuming that the company’s selling price is 140% of inventory cost, the adjusted
cost of Arrow Company’s inventory at December 31, 2012 should be
a. P1,055,000 b. P1,039,500 c. P1,039,300 d. 1,037,3000
37. On December 31, 2012, the cash account of The Voice Company shows the
following composition: (M = million)

Petty cash fund – P0.06M; Cash in bank (payroll fund) – P4M; Travel fund – P0.30M;
Dividend and interest fund – P0.50M; Taxes (e.g., income tax, VAT, EWT, among
others) fund – P0.24M; Cash in bank (current) – P6M; Certificates of deposit (terms
90 days) – P2M; Certificate of deposit (term 180 days) – P3M; Cash in foreign bank
(restricted) – P1M; Money market fund (60 days) – P1M; Money market funds (6
months) –
P1.8M; Customer’s check dated January 14, 2013 – P0.12M; Customer’s check dated
December 30, 2012 returned for lack of funds – P0.08M; A 30-day BSP treasury bill –
P2M; A 3-year BSP treasury bill acquired three months prior to maturity date –
P2.4M; Bond sinking fund – P1.6M; Preferred redemption fund –

P0.8M; Contingent fund – P0.6M; Insurance fund – P0.40M; Fund for acquisition of a
machine – P1M; Traveler’s checks – P0.12M; Cashier’s checks – P0.20M; Savings
deposit set aside for dividends payable on July 31, 2013 – P1M; Pension fund –
P0.80M; Customer’s check outstanding for 18 months – P0.10M; Savings deposit in
closed bank – P0.10M.

What is the correct cash and cash equivalents balance to be reported by The Voice
Company on December 31,
2012?
a. P19.820M b. P18.820M c. P16.420M d. P15.620M

38.Which of the following would not be a correct form for an adjusting entry?
a. A debit to revenue and a credit to liability
b. A debit to an expense and a credit to a liability
c. A debit to a liability and a credit to a revenue
d. A debit to an asset and a credit to a liability

39.Which of the following best defines a prepayment and a deferral?


a. Adjusting entries where cash flow precedes revenue or expense
recognition.
b. Adjusting entries where revenue or expense recognition precedes cash flow.
c. Adjusting entries where cash flow and revenue or expense recognition are
simultaneous.
d. Adjusting entries where revenues or expenses are recognized in the absence
of cash flow.

40.The purpose of the adjusting entries is to

a. Prepare revenue and expense accounts for recording the transaction of the
next period.
b. Apply the realization principle and the matching principle to
transactions affecting two or more accounting periods.
c. Adjust daily the balances in asset, liability, revenue and expense accounts
for the effects of business transactions.
d. Adjust the capital account for the revenue, expense and withdrawal
transactions which occurred during the year.

41.Which of the following properly describes a deferral?


a. Cash is received after revenue is earned.

b. Cash is paid at the same time period that an expense is incurred.


41.An entity owns a number of farms that harvest produce seasonally. What is the
disclosure suggestion if the business is highly seasonal?
 Additional notes should be written in the interim reports about the seasonal
nature.
 Disclosure of financial information for the latest and comparative 12-
month period in addition to the interim report.
 Additional disclosure in the accounting policy note.
 No additional disclosure.

42.Which is true concerning the 75% overall size test for operating segments?
 The total external and internal revenue of all reportable segments is
75% or more of the entity’s external revenue.
 The total external revenue of all reportable segments is 75% or
more of the entity’s external and internal revenue.
 The total external revenue of all reportable segments is 75% or more of the
entity’s external revenue.
 The total internal revenue of all reportable segments is 75% or more of the
entity’s internal revenue.
43. Operations and cash flows that can be clearly distinguished, operationally and for
financial reporting purposes, from the rest of the entity
C. Discontinued
A. Cash-generating unit operations
Component of
B. the entity D. Disposal group
48. The statement of financial position is useful for analyzing all of the
following, except
Profitab
A. Liquidity C. ility
Financia flexibil
B. Solvency D. l ity

 Which ONE of the following statements best describes 'residual value'?


← The estimated net amount currently obtainable if the asset were at the
end of its useful life
← The present value of estimated future cash flows expected to arise from the
continuing use of the asset and from its ultimate disposal
← The amount at which the asset could be exchanged between knowledgeable,
willing parties in an arm's length transaction
← The amount of cash or cash equivalents that could currently be obtained by
selling the asset in an orderly disposal

 Which of the following statements about accounting recognition is (are) true?


← No asset can simultaneously be an asset of more than one entity.
← In accounting, there are instances when a gain or a loss would arise upon initial
recognition of an asset.
← An appropriate basis for recognizing an asset is when a particular enterprise
acquires the right to utilize and control access to the asset’s benefits.
← All of the above statements are true.

 According to the conceptual framework, the following are enhancing qualitative


objectives of accounting except
Reliab Verifiabilit
A. ility C. y
Timelin Comparabi
B. ess D. lity

44.Which is incorrect concerning the qualitative characteristics of financial statements?


← The need for comparability dictates that the seasonality nature of the activities of
an entity be disclosed in interim financial statements.
← The fundamental quality of faithful representation includes the need to present
information that is prudent, neutral, and free from material error.
← When information that is a result of related party transactions are shown
in the financial statements and described as such, these financial
statements lose their reliability.
← Information has the quality of relevance when it influences the economic
decisions of users by helping them evaluate past, present or future events or
confirming or correcting their past evaluations.

45.Compared with other types of money prices used for measuring resources in
financial accounting, the best
argument in favor of historical cost is that it is more
A. Relevant C. Conservative
Object D. Indicative of the entity’s
B. ive purchasing power

54. Financial accounting and reporting focuses on the information needs of


Financial analysts and
A. Management C. consultants
Creditors and
B. investors D. All of the above
 Compared to its 2017 cash basis net income, Despair Company’s 2017 accrual basis
net income increased when it
← Sold used equipment for cash at a gain in 2017
← Recorded less account sales than that of the previous year
← Had more accrued expenses on December 31, 2016than on January 1,
2017
← None of the above

 Which of the following accounting measurements may be made with the greatest
degree of objectivity
Cost of land owned as
A. Net Income C. at year-end
Appraised value of land at
B. Bad debts expense for the year D. year-end

 Which of the following is not part of the “due process” in the financial reporting
standard setting process in the Philippines?
← Publication in the PRC Official Gazette
← Creation of a task force by the standard setting body to study the proposed
accounting standard
J. Distribution of the exposure draft for comment to CPA professionals and other
interested parties
K. None of the above

2 Statement I: The periodicity assumption of accounting (used by the International


Accounting Standards Board) makes depreciation and amortization policies justifiable and
appropriate.
Statement II: The IASB conceptual framework specifically identifies accrual basis
accounting as one of its
fundamental assumptions.
False,
A. True, true C. true
False,
B. True, false D. false

46.Statement I: Almost There, Inc. applies different accounting treatments to similar events
from period to period.
Almost There, Inc. is violating verifiability as described by the International
Accounting Standards Board’s (IASB’s) Conceptual Framework.
Statement II: Under International Financial Reporting Standards (IFRS) companies need
not report immaterial
items within the body of the financial statements, but must disclose them in the notes or
supplementary information that accompany the financial statements.
False,
A. True, true C. true
False,
B. True, false D. false

60. Statement I: Verifiability and predictive value are two ingredients of faithful
representation.
Statement II: The idea of consistency does not mean that companies cannot switch from
one accounting method to another.
False,
A. True, true C. true
False,
B. True, false D. false
The Professional CPA Review School
Main: 3F C. Villaroman Bldg. 873 P. Campa St. cor Espana, Sampaloc, Manila
 (02) 735 8901 / 735 9031 / 0922 861 0191
email add: crc_ace@yahoo.com
Baguio Davao
Rudel Bldg. V, Lower Mabini cor Diego Silang, Baguio 3/F GCAM Bldg. Monteverde St. Davao City
 
City (074) 442-1440 / 0922-8499196 (082) 285-8805 / 0925-7272223

FINANCIAL ACCOUNTING & REPORTING (PROBLEMS) MAY 2019 BATCH

Use the following information for numbers 1 - 2


On December 31, 2016 Jasmine Company factors P450,000 of its receivables to
Genie Company on a with recourse basis. The agreement includes a factoring fee of
8% and a 10% holdback both based on the factored accounts.
Genie Company shall maintain the holdback account at 10% of the uncollected
receivables and will make payments to Jasmine Company at the end of each month
for any excess. Jasmine Company had previously established an Allowance for
Doubtful Accounts for these receivables of P12,000. The recourse obligation has a
fair value of 8,000.

6. The amount of cash received by Jasmine Company from the factoring


a. 369,000 b. 369,960 c. 372,600 d. 377,100
7. The loss arising from the factoring of the receivables
a. 24,000 b. 32,000 c. 35,040 d. 44,000

8. Romania Company purchased from Sweden Company a P20,000, 8%, 5-year note
that required five equal annual year-end payments of P5,009. The note was
discounted to yield a 9% rate to Romania. At the date of purchase Romania
recorded the note at its present value of P19,485.
Total interest revenue earned by Romania over the
life of this note is
d. P
a. P 5,045 b. P 5,560 c. P 8,000 9,000

11. Ohio Company uses the statement of financial position approach in estimating
uncollectible accounts expense. The entity prepares an adjusting entry to
recognize this expense at the end of the year. During the year, Ohio Company
wrote off a P50,000 receivable and made no recovery of previous write-off.
After the adjusting entry for the year, balance in the allowance for doubtful
the credit accounts was
P250,000 larger than it was on January 1. The uncollectible account expense
recorded for the year is
d.
250,00 300,00
a. 50,000 b. 200,000 c. 0 0

11. Grey Company holds an overdue note receivable of P800,000 plus recorded
accrued interest of P64,000. The effective interest rate is 8%. As a result of a
court-imposed settlement on December 31, 2016, Grey agreed to the following
restructuring arrangement:
← Reduced the principal obligation to P600,000.
← Forgave the P64,000 of accrued interest.
← Extended the maturity date to December 31, 2018.
← Annual interest of P40,000 is to be paid on December 31, 2017 and 2018.
The present value of the interest and principal payments to be received by
Grey Company discounted for two years at 8% is P585,734. On December 31,
2016, Grey would recognize a valuation allowance for impaired loans of
a. 278,266 b. 214,266 c. 198,266 d. 54,266

16. On March 1, 2016, the Heisman Company recorded two sales of P20,000 and P30,000 under credit terms of 3/10, n/30.
Payment for the P20,000 sale was received March 11 Payment for the P30,000 sales was received on March 15. Under the
gross method and net method, net sales in the March 2016 income statement are
Gross Net Gross Net
method method method method
P P
c 49,40 48,50
a. P48,500 P48,500 . 0 0
P
49,40 P49,4
b. P48,500 P49,500 d. 0 00

18. On December 31, 2016, Bears Company sold a machine to Chicago Company in
exchange for a noninterest bearing note requiring ten annual payments of
P100,000. Chicago Company made the first payment on December 31, 2016. The
market interest rate for similar notes at date of issuance was 8%.
Interest income to be reported in
the 2018
d.
a. 80,000 b. 58,982 c. 49,651 45,973
Page 2

19. The Receivable control account of Florida Company shows a balance of P114,000
as of
Accounts receivable from regular customers P52,000
Advance to creditors on purchase orders 15,000
Customers' credit balances (5,000)
Notes receivable dishonored, charged back to accounts
receivable 4,000
Selling price of goods shipped to customers on December
31, 2016
under terms FOB shipping point 8,000
Subscription receivable, due June 30, 2017 40,000
The correct balance of current trade accounts receivable of Florida Company as
of December 31, 2016 is
a. 56,000 b. 60,000 c. 64,000 d. 69,000

3 On December 31, 2016, Boston Company sold used equipment and received a
noninterest-bearing note requiring payment of P500,000 annually for ten years.
The first payment is due December 31, 2016 and the
prevailing rate of interest for this type of note at date of issuance is 12%.
Present value factors are as follows:
Present value of 1 at 12% for 10
periods 0.32
Present value of ordinary annuity of 1 at 12%
for 10 periods 5.65
The unearned interest income at December 31, 2016
is
1,836,
a. 2,336,000 b. 000 c. 1,536,000 d. 1,516,000

Use the following for numbers 10 -11


On December 1, 2016 Packers Company assigned P1,500,000 of accounts receivable
to Goo2 Company as a security for a loan of P900,000 which carries an interest rate
of 10%.Packers Company charged a 0.5% commission on the amount of the loan.
During December, Packers Company collected P220,000 on the assigned accounts
after deducting discounts P1,500 of discounts. Packers Company accepted returns of
P2,700 and wrote off assigned accounts totaling P3,000.
The amount of cash received arising from the assignment
10. of the receivables is
b 895,5
a. 888,000 . 890,625 c. 892,500 d. 00
11. The carrying amount of the accounts receivable assigned is
a b 1,280,0
. 1,272,800 . 1,274,300 c. 1,278,500 d. 00
20. Potter Corporation provides an allowance for its doubtful accounts receivable. At
December 31, 2015, the allowance account had a credit balance of P8,000. Each
month Potter accrues bad debts expense in an amount equal to 2% of credit
sales. Total credit sales during 2016 amounted to P2,000,000. During 2016
uncollectible accounts receivable totaling P22,000 were written off against the
allowance account. An aging of accounts receivable at December 31, 2016
indicates that an allowance of P42,000 should be provided for doubtful accounts
as of the date.
Accordingly, bad debts expense previously accrued during 2016 should be
increased by
a. 62,000 b. 56,000 c. 42,000 d. 16,000

21. On July 1, 2015, Capitols Company sold equipment to WSTN Company for
P250,000. Capitols Company accepted a 10% note receivable for the entire sales
price. This note is payable in two equal installments of P125,000 plus accrued
interest on December 31,2015, and December 31,2016. On July 1, 2016, Capitols
Company discounted the note at a bank at an
interest rate of 12%.
The cash received by Capitols Company from the note
discounting is
d.
a. 246,750 b. 233,500 c. 129,250 123,375

23. On January 1, 2016, Broncos Company sold goods to Denver Company. Portland
signed a noninterest-bearing note requiring payment of P600,000 annually for
seven years. The first payment was made on
January 1, 2016. The prevailing rate for this type of note
at issuance date was 10%.
The amount recorded as sales revenue in January
2016 is
3,216, d 2,142,0
a. 000 b. 2,922,000 c. 2,616,000 . 00

e. On June 1, 2016, Apple Company sold merchandise with a list price of P200,000
to Microsoft Company on account. Apple Company allowed trade discounts of
20% and 10%. Credit terms were 2/10, n/30 and sale was made FOB shipping
point. Apple Company prepaid P3,000 of insurance for Microsoft Company as an
accommodation.
On June 11, 2016, Microsoft Company paid Apple
Company
d.
a. 147,000 b. 144,120 c. 144,060 141,120
Page 3

3 49ers Company factored P6,000,000 of its accounts receivables. The transfer is


recorded as a sale by SF Company. SF Company retained 5% for sales
adjustments and charged a P120,000 finance fee
The loss to be reported as a result of the transfer is
d.
a. 0 b. 120,000 c. 300,000 420,000

25. Michigan Company prepared an aging of its accounts receivable at December


31, 2016 and determined
that the net realizable value of the receivables at that date was P50,000.
Additional information is available as follows:
Accounts receivable, 12/31/2015 P 48,000
Accounts receivable, 12/31/2016 54,000
Allowance for doubtful accounts at 12/31/2015
(credit balance) 6,000
Accounts written off as uncollectible during 2016 5,000
Michigan Company’s bad debt
expense for 2016 is
3,0
a. 00 b. 4,000 c. 5,000 d. 7,000

 MS Company showed the following information related to the accounts


receivable in order to estimate bad
debts through the use of the aging. The credit period of the company is 30 days
on the average.
Age of Receivables Amount
Under 30 P4,000,
days 000
60
31 - days 1,500,000
day
61 - 90 s 1,000,000
- day
91 120 s 500,000
- day
121 150 s 200,000
- day
151 180 s 100,000

Accounts which are overdue for less


than 30 days 96%
Accounts which are
overdue 31 – 60 days 90%
Accounts which are overdue 61 – 90
days 75%
Accounts which are
overdue 91 – 120 days 55%
Accounts which are overdue 121 – 150
days 35%
Accounts which are overdue for over
150 days 5%
What is the net realizable value of accounts
receivable?
c. d.
a. b. P6,385,00 P6,860,0
P6,135,000 P6,290,000 0 00

27. On July 1, 2016, Target Company obtained a two-year 10% note receivable for
services rendered. At that time, the market rate of interest was 12%. The face
amount of the note and the entire amount of interest are due on June 30, 2018.
Interest receivable on December 31, 2016 is
a. 5% of the face amount of the note
b. 6% of the face amount of the note
c. 5% of the July 1, 2016 present value of the note due on June 30, 2018
d. 6% of the July 1, 2016 present value of the note due on June 30, 2018

28. On July 1 of the current year, Krogers Company received a one-year note
receivable bearing interest at the market rate. The face amount of the note
receivable and the entire amount of the interest are due on June 30 of next year.
On December 31 of the current year, Krogers Company should report in the
statement of financial position
a. A deferred credit for interest applicable to next year
b. No interest receivable
c. Interest receivable for the entire amount of the interest due on June 30 of
next year
d. Interest receivable for the interest accruing in the current year

29. Tresh, Inc. had the following bank reconciliation at March 31, 2015:
P37,2
Balance per bank statement, 3/31/15 00
15,30
Add: Deposit in transit 0
47,50
0
12,60
Less: Outstanding checks 0
P34,9
Balance per books, 3/31/15 00
Data per bank for the month of April 2015
follow:
Deposits P46,700
Disbursements 49,700
Page 4

All reconciling items at March 31, 2015 cleared the bank in April. Outstanding
checks at April 30, 2015 totaled P6,000. There were no deposits in transit at
April 30, 2015. What is the cash balance per books at April 30, 2015?
a. P28,200 b. P31,900 c. P34,200 d. P38,500

g. Wellington Corp. has outstanding accounts receivable totaling P6.5 million as of


December 31 and sales on credit during the year of P24 million. There is also a
credit balance of P12,000 in the allowance for doubtful accounts. If the company
estimates that 8% of its outstanding receivables will be uncollectible,
what will be the amount of bad debt expense
recognized for the year?
d. P
a. P 532,000. b. P 520,000. c. P1,920,000 508,000.

 Kaniper Company has the following items at year-end:


Cash in
bank P20,000
Petty cash 300
Commercial paper with maturity of 2
months 5,500
Post-dated checks 1,400
Kaniper should report cash and cash
equivalents of
c.P25,
a. P20,000 b. P20,300 800 d. P27,200

29. Rosalie Co. uses the gross method to record sales made on credit. On June 15,
2016, it made sales of P150,000 with terms 2/15, n/30 to Finley Farms, Inc. On
June 19, 2016, Rosalie received payment for 1/2 the amount due from Finley
Farms. Rosalie's fiscal year end is on June 30, 2016. What amount will be
reported in the statement of financial position for the accounts receivable due
from Finley Farms, Inc.?
a. P49,000 b. P50,000 c. P48,000 d. P51,000

39. Vasguez Corporation had a 1/1/15 balance in the Allowance for Doubtful
Accounts of P20,000. During 2015, it wrote off P14,400 of accounts and collected
P4,200 on accounts previously written off. The balance in Accounts Receivable
was P400,000 at 1/1 and P480,000 at 12/31. At 12/31/15, Vasguez
estimates that 5% of accounts receivable will prove to be uncollectible. What is
Bad Debt Expense for 2015?
a. P4,000 b. P14,200 c. P18,400 d. P24,000

34. If a petty cash fund is established in the amount of P250, and contains P150 in
cash and P95 in receipts for disbursements when it is replenished, the journal
entry to record replenishment should include credits
to the following accounts
c. Cash, P95; Cash Over and
a. Petty Cash, P75. Short, P5.
d. Cash,
b. Petty Cash, P150 P150.

37. If the month-end bank statement shows a balance of P36,000, outstanding


checks are P12,000, a deposit of P4,000 was in transit at month end, and a check
for P500 was erroneously charged by the bank against
the account, the correct balance in the bank
account at month end is
d.
P43,500
a. P27,500 b. P28,500 c. P20,500 .

38. Finley, Inc.’s checkbook balance on December 31, 2015 was P21,200. In
addition, Finley held the following items in its safe on December 31.
a. A check for P450 from Peters, Inc. received December 30, 2015, which was
not included in the checkbook balance.
b. An NSF check from Garner Company in the amount of P900 that had been
deposited at the bank, but was returned for lack of sufficient funds on
December 29. The check was to be redeposited on January 3, 2016. The
original deposit has been included in the December 31 checkbook balance.
c. Coin and currency on hand amounted to P1,450.
The proper amount to be reported on Finley's statement of financial position for
cash at December 31, 2015 is
d.
P21,750
a. P21,300 b. P20,400 c. P22,200 .
43. On June 1, 2015, Nott Corp. loaned Horn £400,000 on a 12% note, payable in
five annual installments of P80,000 beginning January 2, 2016. In connection
with this loan, Horn was required to deposit P5,000 in a noninterest-bearing
escrow account. The amount held in escrow is to be returned to Horn after all
principal and interest payments have been made. Interest on the note is payable
on the first day of each month beginning July 1, 2015. Horn made timely
payments through November 1, 2015. On January 2, 2016, Nott received
payment of the first principal installment plus all interest due. At December 31,
2015,

d.
c.P8,0 P12,000
a. P0. b. P4,000 00. .
Page 5

44. In preparing its bank reconciliation for the month of April 2015, Henke, Inc. has
available the following information.
Balance per bank statement,
4/30/15 P39,140
NSF check returned with 4/30/15 bank
statement 450
Deposits in transit, 4/30/15 5,000
Outstanding checks, 4/30/15 5,200
Bank service charges for
April 20
What should be the correct balance of cash at
April 30, 2015?
b.P38,
b. P39,370 940 c. P38,490 d. P38,470

 The cash account shows a balance of P45,000 before reconciliation. The bank
statement does not include a deposit of P2,300 made on the last day of the
month. The bank statement shows a collection by the bank of P940 and a
customer's check for P320 was returned because it was NSF. A customer's check
for P450 was recorded on the books as P540, and a check written for P79 was
recorded as P97. The correct balance
in the cash account was
c. P45,512 b. P45,548 c. P45,728 d. P47,848.

46. At the end of 2016, Diego Company made four adjusting entries for the following
items:
← Depreciation expense, RP25,000.
← Expired insurance, RP2,200 (originally recorded as prepaid insurance.)
← Interest payable, RP6,000.
← Rental revenue receivable, RP15,000.
In the normal situation, to facilitate subsequent entries, the adjusting entry or
entries that may be reversed is (are)
c. Entry No. 3 and
a. Entry No. 3. No. 4.
d. Entry No. 2, No. No.
b. Entry No. 4. 3 and 4.

 Eaton Co. sells major household appliance service contracts for cash. The
service contracts are for a one-year, two-year, or three-year period. Cash receipts
from contracts are credited to Unearned Service Revenues. This account had a
balance of P1,800,000 at December 31, 2015 before year-end adjustment.
Service contract costs are charged as incurred to the Service Contract Expense
account, which had a balance of P450,000 at December 31, 2015. Service
contracts still outstanding at December 31, 2015
expire as follows:
During P380,0
2016 00
During 570,00
2012 0
During 350,00
2013 0
What amount should be reported as Unearned Service Revenues in Eaton's
December 31, 2015 statement of financial position?
a. P1,350,000. b. P1,300,000. c. P850,000 d. P500,000.

 Jim Yount, M.D., keeps his accounting records on the cash basis. During 2016,
Dr. Yount collected Rp 360,000 from his patients. At December 31, 2015, Dr.
Yount had accounts receivable of Rp 50,000. At
December 31, 2016, Dr. Yount had accounts receivable of Rp 70,000 and
unearned revenue of Rp 15,000. On the accrual basis, how much was Dr. Yount's
patient service revenue for 2016?
a. P 315,000. b. P c. P 380,000. d. P
370,000. 390,000.

45. On the December 31, 2015 statement of financial position of Vanoy Co., the
current receivables consisted of the following:
Trade accounts receivable P 75,000
Allowance for uncollectible
accounts (2,000)
Claim against shipper for goods lost in transit
(November 2015) 3,000
Selling price of unsold goods sent by Vanoy on consignment at
130% of cost
(not included in Vanoy 's ending
inventory) 26,000
Security deposit on lease of warehouse used for storing some
inventories 30,000
P132,0
Total 00
At December 31, 2015, the correct total of Vanoy's current net
receivables was
P
76,00 d.
a. 0. b. P152,000 c. P156,000 P132,000.
/mft
CRC-ACE
The Professional CPA Review School

Main: 3F C. Villaroman Bldg. 873 P. Campa St. cor Espana, Sampaloc, Manila  (02) 735
8901 / 735 9031
Branch: Rudel Bldg. V, Lower Mabini cor Diego Silang, Baguio City  (074) 442-1440
3/F GCAM Bldg. Monteverde St. Davao City  (082) 285-8805/ 0933-5470716
DRILL – 1 OCTOBER 2018
www.crcace.com email add: crc_ace@yahoo.com
FULL TIME REVIEW

FINANCIAL ACCOUNTING AND REPORTING (PROBLEMS)

1. On January 1, 2012, the capital of Mila Company was P2,600,000 and on December 31, 2012, the
capital was P3,600,000. During the current year, Mila, the owner, withdrew merchandise with
carrying amount of P150,000 which were marked to sell for P230,000, and paid a P400,000 note
payable of the business with interest of 10% for six months with a check drawn on a personal
checking account.
The net income for 2012 is
a. 730,000 b. 810,000 c. 1,190,000 d. 1,270,000

2. An analysis of the records of Alden Company disclosed changes in account balances for the
current year and the supplementary data listed below:
Cash 600,000 increase
Accounts receivable 350,000 decrease
Merchandise inventory 1,320,000 increase
Accounts payable 560,000 increase
Accrued expenses 90,000 decrease
During the year Alden borrowed P2,000,000 from the bank and paid off P1,750,000 plus interest of
P200,000. Interest of P50,000 is accrued on December 31. There was no interest payable at the beginning
of the year.
Furthermore, Alden transferred equity securities to the business which were sold for P900,000 to finance the
acquisition of merchandise. Alden made weekly withdrawals in the current year of P15,000.
The net income for the current year is
a. 580,000 b. 680,000 c. 730,000 d. 930,000

3. The trial balance of HI Company does not balance. The debit column totaled P588,600 while the
credit column totaled P598,600. An examination of the ledger shows these errors
 Cash received from customer on account was recorded (both debit and credit) as P46,900 instead of
P49,600
 Check issued to supplier was recorded (both debit and credit) as P24,800 instead of P28,400
 The purchase on account of a computer table worth P22,000 was recorded as a debit to Office
Expense and a credit to accounts payable.
 Services performed on account for a client, for P12,250 were recorded both as a debit to Accounts
receivable and a credit to service revenue of P21,250
 A payment of P2,500 for telephone charges was posted as a credit to Office Expense and a credit to
Cash
 The unearned rent account was totaled at P15,200 instead of P12,200
 The debit footings to purchases and interest income were both understated by P1,000
The correct debit/credit column totals should be
a. 582,000 b. 591,000 c. 600,000 d. 607,200

4. The trial balance prepared at December 31 did not balance. Dr total was P159,250 and Cr total
was P153,200. In determining the cause of the difference, you discovered the following errors: a
credit to cash of P650 was not posted; a P2,000 credit to be made to the Sales account was
credited to the Accounts receivable account instead; the wages payable account balance of
P9,300 was listed in the trial balance as P3,900.
The correct trial balance is
a. 153,200 b. 159,250 c. 160,600 d. 161,250
5. The following errors were made in preparing a trial balance, the P1,350 balance of inventory was
omitted; the P450 balance of Prepaid Insurance was listed as a credit; and the P300 balance of
Salaries Expense was listed as Utilities Expense.

The dr/cr totals of the trial balance would differ by


a. P1,350 b. P1,800 c. P2,100 d. P2,250

Use the following information for questions 6 - 8:


Excess Company paid or collected during 2013 the following items:
Insurance premiums paid P 25,450; Interest collected P 53,750;
Salaries paid P 116,300

The following balances were from Excess Company’s balance sheets:


December 31, 2012 December 31, 2013
Prepaid insurance P 2,400 P 1,700
Interest receivable 4,700 3,200
Salaries payable 12,300 10,600

6. The insurance expense on the income statement for 2013 was


a. 25,450 b. 26,150 c. 24,750 d. 23,750
7. The interest revenue on the income statement for 2013 was
a. 53,750 b. 55,250 c. 52,250 d. 50,550
8. The salary expense on the income statement for 2013 was
a. 116,300 b. 118,000 c. 114,600 d. 105,700

Use the following data for nos. 9 - 10


PS Company’s trademark was licensed to FC Company for royalties of 15% of sales of the
trademarked items. Royalties are payable semi-annually on March 15 for sales in July-December
of the prior year, and on September 15 for sales in January-June of the same year. PS Company
received the following royalties from FC Company:
March 15 September 15
2012 P10,000 P15,000
2013 12,000 17,000
Estimated sales of trademarked items for July-December 2013 is P60,000

9. Under cash basis, PS Company would report royalty revenues for 2013 of
a. P26,000 b. P29,000 c. P38,000 d. P41,000
10. Under accrual method, PS Company would report royalty revenues for 2013 of
a. P26,000 b. P29,000 c. P38,000 d. P41,000

11. On January 1 of the current year a group of stockholders set up AB Corporation. They
contributed cash of P4,250,000 and borrowed P950,000. During the year, revenues from sales
totaled P1,400,000, while total costs and expenses were P750,000. AB Corporation declared a
cash dividend of P300,000 on December 20, payable to the stockholders on January 30 of the
following year. There were no additional activities affecting stockholders’ equity. By December
31 of current year, liabilities decreased to P880,000.

Total assets at the end of the current year is


a. 4,670,000 b. 5,480,000 c. 5,780,000 d. 6,430,000

12. The assets of BC Company amounted to P70,000 on January 1, but increased by P80,000 by
December 31. During this same period liabilities decreased to P20,000. The owner’s equity on
January 1, amounted to P35,000.
The amount of owner’s equity at December 31 is
a. 60,000 b. 65,000 c. 130,000 d. 135,000

13. FG Corporation’s account balances during the year showed the following changes: increases
(decreases)
Current assets P 278,000 Non-current assets (P 97,000)
Current liabilities 54,000 Non-current liabilities ( 20,000)
Ordinary shares 60,000 Additional paid-in capital 12,000
Revaluation surplus 50,000 Treasury shares 10,000

There were no changes in retained earnings for the year other than a P30,000 dividend payment
and the year’s net earnings. FG Corporation’s reported net income is
a. 15,000 b. 35,000 c. 45,000 d. 65,000
14. On January 1, 2013, the capital of Corinthians Company was P 1,700,000 and on December 31,
2013, the capital was P 2,400,000. During the current year, Corinthians withdrew merchandise
costing P 100,000 and with sales value of P 180,000, and paid a P 1,000,000 note payable of the
business with interest of 12% for six months with a check drawn on a personal checking
accounting.
The net income (loss) for 2013 is
a. 260,000 b. (260,000) c. 180,000 d. (180,000)

15. On January 1, 2013, the statement of financial position of Richway Company showed total assets
of P 5,000,000 total liabilities of P 2,000,000 and contributed capital of P 2,000,000. During the
current year, the corporation issued share capital of P 500,000 par value at a premium of
P 300,000. Dividend of P 250,000 was paid on December 31, 2013. The statement of financial
position on December 31, 2013 showed total assets of P 7,500,000 and total liabilities of
P3,200,000.
The net income for the current year is
a. 1,750,000 b. 1,000,000 c. 750,000 d. 500,000

16. Dec Company uses the statement of financial position approach in estimating uncollectible
accounts expense. The company prepares an adjusting entry to recognize this expense at the
end of each month. During the month of July, the company wrote off a P 1,000 receivable and
made no recoveries of previous write-offs. Following the adjusting entry for July, the credit
balance in the Allowance for Doubtful Accounts was P 2,500 larger than it was on July 1.
What amount of uncollectible account expense was recorded for July?
a. P 2,500 b. P 1,000 c. P 1,500 d. P 3,500

17. Based on the information:


Credit sales P 172,000 credit
Collections on accounts receivable during the year 170,000 credit
Cash sales 810,000 credit
Unadjusted balance in Allowance for doubtful accounts 50 debit
Sales returns and allowances for credit sales 4,000 credit
Accounts receivable, beginning of the year 14,000 credit
If bad debts are estimated to be 1 ½% of ending accounts receivable, in the adjusting entry to
recognize bad debts, you would debit bad debt expense for:
a. P 230 b. P 190 c. P 130 d. P 180

18. The credit total of a trial balance is greater than the debit total of P1,800. Which of the following
is the most likely cause of the error?
a. A debit amount of P7,950 was erroneously recorded as P9,750
b. A debit amount of P1,800 was erroneously entered in the credit side of the trial balance
c. A credit amount of P9,750 was erroneously recorded as P7,950
d. A debit amount of P9,750 was erroneously recorded as P7,950

19. The trial balance of a company allowed a debit total which exceeded the credit total by P8,100. Which of
the following is probably responsible for this situation?
a. The balance of P89,000 in Accumulated Depreciation was entered in the trial balance as
P80,900
b. The balance of P4,050 in advertising expense was entered as a credit in the trial balance
c. The balance of P9,000 in Office Equipment was entered as a debit of P900 in the trial balance
d. The balance of P16,200 in Unexpired Insurance was entered as a credit in the trial balance

20. The capital account of Thirst Company decrease by P15,000 in 2013. During the year, Jim Water, the owner
issued a personal check to settle Thirst Company’s obligation amounting to P20,000. At the end of the year,
Jim Water took merchandise costing P10,000 for personal use. At year-end, Thirst Company’s net income
(loss) is
a. 5,000 b. (5,000) c. 25,000 d. (25,000)

21. The accounts in the ledger of CD Company contain the following balances at yearend: Accounts
receivable, P30,240; Cash, P50,985; Equipment, P172,760; Gas and oil expense, P2,650;
Insurance expense, P1,830; Notes payable, P64,575; Prepaid insurance, P6,880; Repair expense,
P3,360; Service revenue, P37,130; CD drawing, P2,450; CD capital (beg), P156,290; Salaries
expense, P15,490; Salaries payable, P2,850. Assuming no error committed during the fiscal
period, the balance of Accounts payable is
a. 20,900 b. 23,800 c. 25,800 d. 31,500

22. John Company received cash of P77,400 on September 1, 2012 for one year's rent in advance
and recorded the transaction with a credit to a nominal account. The December 31, 2012
adjusting entry is
a. Dr Rent revenue; Cr Unearned rent P25,800
b. Dr Unearned rent; Cr Rent revenue, P25,800
c. Dr Rent revenue; Cr Unearned rent P51,600
d. Dr Unearned rent; Cr Rent revenue P51,600

23. On March 31, the ledger for GH Services consists of the following: Cleaning equipment, P27,800;
Accounts payable, P15,700; Gail, capital, P20,000; Office equipment, P11,500; Accrued interest
on note, P1,500; Cleaning supplies, P2,600; Accounts receivable, P21,000; Accumulated
depreciation, P2,000; Cash, P7,900; Note payable, P22,000; Accrued salaries P9,600. In a trial
balance prepared on March 31, the total of the credit column is
a. 68,800 b. 69,300 c. 70,800 d. 72,300

24. The trial balance of HI Company does not balance. The debit column totaled P588,600 while the
credit column totaled P598,600. An examination of the ledger shows these errors
Cash received from customer on account was recorded (both debit and credit) as P46,900 instead of
P49,600
 Check issued to supplier was recorded (both debit and credit) as P24,800 instead of P28,400
 The purchase on account of a computer table worth P22,000 was recorded as a debit to Office
Expense and a credit to accounts payable.
 Services performed on account for a client, for P12,250 were recorded both as a debit to Accounts
receivable and a credit to service revenue of P21,250
 A payment of P2,500 for telephone charges was posted as a credit to Office Expense and a credit to
Cash
 The unearned rent account was totaled at P15,200 instead of P12,200
 The debit footings to purchases and interest income were both understated by P1,000
The correct debit/credit column totals should be
a. 582,000 b. 591,000 c. 600,000 d. 607,200

25. The following errors were made in preparing a trial balance, the P1,350 balance of inventory was
omitted; the P450 balance of Prepaid Insurance was listed as a credit; and the P300 balance of
Salaries Expense was listed as Utilities Expense. The dr/cr totals of the trial balance would differ
by
e. P1,350 f. P1,800 g. P2,100 h. P2,250

26. The trial balance of HI Company does not balance. The debit column totaled P588,600 while the
credit column totaled P598,600. An examination of the ledger shows these errors
 Cash received from customer on account was recorded (both debit and credit) as P46,900 instead of
P49,600
 Check issued to supplier was recorded (both debit and credit) as P24,800 instead of P28,400
 The purchase on account of a computer table worth P22,000 was recorded as a debit to Office
Expense and a credit to accounts payable.
 Services performed on account for a client, for P12,250 were recorded both as a debit to Accounts
receivable and a credit to service revenue of P21,250
 A payment of P2,500 for telephone charges was posted as a credit to Office Expense and a credit to
Cash
 The unearned rent account was totaled at P15,200 instead of P12,200
 The debit footings to purchases and interest income were both understated by P1,000
The correct debit/credit column totals should be
a. 582,000 b. 591,000 c. 600,000 d. 607,200

27. On January 1, 2013, the statement of financial position of Richway Company showed total assets
of P 5,000,000 total liabilities of P 2,000,000 and contributed capital of P 2,000,000. During the
current year, the corporation issued share capital of P 500,000 par value at a premium of
P 300,000. Dividend of P 250,000 was paid on December 31, 2013. The statement of financial
position on December 31, 2013 showed total assets of P 7,500,000 and total liabilities of
P3,200,000.
The net income for the current year is
b. 1,750,000 b. 1,000,000 c. 750,000 d. 500,000

28. Dec Company uses the statement of financial position approach in estimating uncollectible
accounts expense. The company prepares an adjusting entry to recognize this expense at the
end of each month. During the month of July, the company wrote off a P 1,000 receivable and
made no recoveries of previous write-offs. Following the adjusting entry for July, the credit
balance in the Allowance for Doubtful Accounts was P 2,500 larger than it was on July 1. What
amount of uncollectible account expense was recorded for July?
a. P 2,500 b. P 1,000 c. P 1,500 d. P 3,500
29. Based on the information:
Credit sales P 172,000 credit
Collections on accounts receivable during the year 170,000 credit
Cash sales 810,000 credit
Unadjusted balance in Allowance for doubtful accounts 50 debit
Sales returns and allowances for credit sales 4,000 credit
Accounts receivable, beginning of the year 14,000 credit
If bad debts are estimated to be 1 ½% of ending accounts receivable, in the adjusting entry to recognize
bad debts, you would debit bad debt expense for:
a. P 230 b. P 190 c. P 130 d. P 180
30. The accounts in the ledger of CD Company contain the following balances at yearend: Accounts
receivable, P30,240; Cash, P50,985; Equipment, P172,760; Gas and oil expense, P2,650; Insurance
expense, P1,830; Notes payable, P64,575; Prepaid insurance, P6,880; Repair expense, P3,360; Service
revenue, P37,130; CD drawing, P2,450; CD capital (beg), P156,290; Salaries expense, P15,490; Salaries
payable, P2,850. Assuming no error committed during the fiscal period, the balance of Accounts payable is
a. 20,900 b. 23,800 c. 25,800 d. 31,500

END OF EXAMINATION

The Professional CPA Review School


Main: 3F C. Villaroman Bldg. 873 P. Campa St. cor Espana, Sampaloc, Manila  (02) 735 8901 / 735 9031
Branch: Rudel Bldg. V, Lower Mabini cor Diego Silang, Baguio City  (074) 442-1440
3/F GCAM Bldg. Monteverde St. Davao City  (082)285-8805 / 0925-7272223
cr

FINANCIAL ACCOUNTING &www.crcace.com


REPORTINGemail add: crc_ace@yahoo.com MAY 2019
BATCH
FIRST PRE-BOARD EXAMINATION FEBRUARY 10; 11:30 –
2:30 PM

INSTRUCTIONS: Select the correct answer for each of the following questions. Mark only one
answer for each item by Shading the corresponding letter of your choice on the answer sheet
provided. STRICTLY NO ERASURES ALLOWED. Use Pencil No. 2 only.
Use the following for items 1 - 3
Canary Company’s Cash in Bank items as of December 31, 2018 included the
following items:
Cash in Bank – BDO checking account P 1,800,000
Cash in Bank – BPI checking account 1,600,000
Cash in Bank - MBTC checking account (per bank statement) 1,400,000
Additional information in relation to the above-mentioned components is as follows:
 The following items were noted in relation to the BDO checking account:
Check No.123543 written and dated on December 27, 2018 in the amount of P60,000
remains at hand as of December 31, 2018
Check No. 123550 written and dated on December 28, 2018 in the amount of P80,000
was picked up at December 31, 2018 but has remained outstanding until January 6,
2018
Check No. 123556 written on December 29, 2018 in the amount of P100,000 dated
January 2, 2019 was picked up at December 31, 2018
 The following information is in relation to Canary Company’s BPI checking accounts:
BPI checking account #10001 P 1,900,000
BPI checking account #10002 (300,000)
Total P 1,600,000
The overdraft in BPI checking account #10002 was due to a check for P120,000 dated
January 2, 2019 and was claimed by the payee on December 29, 2018.
A compensating balance was being maintained in BPI checking account #10001 for
P200,000 in relation to a long-term loan which was restricted as to withdrawal
 The following items were identified in relation to the MBTC checking account; Deposit in
transit at December 31, 2018, P125,000; outstanding checks at December 31, 2018,
P90,000; Service charge for December, P6,000; Proceeds of note sent to MBTC for
collection on December 6, 2018, P35,000; Erroneous bank debit for P15,000; Book error in
recording a deposit of P12,000 as P21,000.
1. The cash in bank – BDO amount reported in the December 31, 2018 statement of financial
position is
a. 1,920,000 b. 1,940,000 c. 1,960,000 d. 2,040,000
2. The cash in bank – MBTC amount reported in the December 31, 2018 statement of financial position is
a. 1,450,000 b. 1,435,000 c. 1,426,000 d. 1,420,000
3. The cash in bank – BPI amount reported in the December 31, 2018 statement of financial
position is
a. 1,400,000 b. 1,520,000 c. 1,600,000 d. 1,700,000

Use the following for items 4 - 7


Wonder Woman Company is selling its condominium unit with a carrying amount of P3,500,000.
It received the following offers and would conduct the sale on January 1, 2019.
 Ivy Company offers to issue a P4,800,000, 3-year note having a stated rate of 9% which was
the prevailing rate. Furthermore, the note shall be collected in three equal amounts to include
the interest that will start one year from the date of the issuance of the note
 Vixen Company offers to issue a P7,500,000, 4-year note. The note shall be collected in equal
amounts which will start at the date of sale. No interest was stipulated. The prevailing market
rate was 9%.
4. The annual collections from the note to be issued by Ivy Company is
a. 1,600,000 b. 1,739,691 c. 1,817,977 d. 1,896,263
5. The gain on sale that Wonder Woman would recognize if it chooses to sell the unit to Vixen
Company
a. 4,000,000 b. 3,121,177 c. 2,847,826 d. 2,574,475
6. The interest income to be reported by Wonder Woman Company in 2020 if it chooses to sell
the unit to Ivy Company
a. 156,573 b. 261,339 c. 300,216 d. 314,308
7. The carrying amount of the note to be reported in Wonder Woman Company’s December 31,
2020 balance sheet if it chooses to sell the unit to Vixen Company
a. 3,298,333 b. 3,595,183 c. 3,763,933 d. 4,046,703

8. Mera Company had the following information relating to its accounts receivable at December
31, 2017 and for the year ended, December 31, 2018
Accounts receivable, December 31, 2017 975,000
Allowance for doubtful accounts, December 31, 2017 117,000
Credit sales for 2018 2,600,000
Discounts granted to credit customers 115,000
Sales returns on credit sales 45,000
Cash collections from credit customers for 2018, including P40,000 from
previously written-off accounts 2,725,000
Accounts written-off in October 2018 60,000
The lifetime expected credit loss of Mera Company was 12%.
The receivable amount reported in the December 31, 2018 statement of financial position of
Mera Company is
a. 670,000 b. 630,000 c. 589,600 d. 554,400

Use the following for items 9 - 12


The following transactions of Zatanna Company during the last quarter of 2018
and January 2019 in relation to its receivables were as follows:
 October 1: Zatanna Company factors P500,000 of its receivables to Dawnstar
Company on a with recourse basis. The agreement includes a factoring fee of 1.5%
and a 4% holdback both based on the factored accounts. Zatanna Company
transferred substantially all the risks and rewards of ownership and the receivable
records are transferred to Dawnstar Company on October 9, 2018. The fair value of
the recourse obligation was P12,000
 November 1: Zatanna Company assigned, under guarantee specific accounts totaling
to P1,800,000 to Isis Finance on a notification basis. Isis Finance advances 85% of
the accounts assigned less a service charge of 1% based on the total accounts
assigned.
 November 30: Zatanna received a statement that Isis Finance collected P780,000 of
the accounts. The November charges which is 1% of the accounts assigned
outstanding as of November 30 shall be deducted from the remittance that will be
due to Zatanna Company
 December 1: Zatanna Company discounted an P800,000, 8-month, 9% note
receivable dated October 1, 2018. The bank discounted the note at 12%.
 December 31: Zatanna Company received a second statement that Isis Finance
collected P550,000 of the accounts. The December charges which is 1% of the
accounts assigned outstanding as of December 31 shall be deducted from the
remittance that will be due to Zatanna Company
 January 8: Received a statement and a check from Dawnstar Company indicating that
all receivables factored were totally collected.
 January 31: Zatanna received a third statement and a check. The stateme indicated
that Isis Company collected an additional P350,000 and made an additional charge of
1% of the accounts assigned outstanding as of January 31.
9. The net proceeds from the receivable financing activities of Zatanna Company is
a. 2,781,620 b. 2,769,620 c. 2,799,620 d. 2,761,620
10. The total loss arising from the receivable financing activities of Zatanna Company in 2018 is
a. 34,380 b. 40,380 c. 52,380 d. 70,380
11. The equity over the assigned accounts included as a note disclosure information in the 2018
financial statement of Zatanna Company is
a. 237,100 b. 241,800 c. 252,000 d. 270,000
12. The amount of the check received from Isis Finance on January 31, 2019 is
a. 115,900 b. 117,100 c. 133,900 d. 135,100

13. Barbara Bank loaned Gordon Company P5,000,000 on January 1, 2016. The terms of the loan
were payment in full on January 1, 2019 plus annual interest payment at 9%. The interest
payment was made as scheduled on January 1, 2017.
However, due to financial setbacks, Gordon Company was unable to make its 2018 interest
payment. Barbara Bank considers the loan impaired and projects the cash flows from the loan
on December 31, 2018. Barbara Bank accrued the interest at December 31, 2017, but did not
continue to accrued interest for 2018 due to the impairment of the loan. The projected cash
flows are as follows: December 31; 2019, P1,000,000; 2020, P1,500,000; 2021, P1,500,000;
2022, P2,000,000
The loan impairment loss on December 31, 2018
a. 1,144,923 b. 716,966 c. 694,923 d. 266,966

Use the following for items 14 - 15


On January 1, 2018 Big Barda Company sold a plot of land and received in exchange a note
having a face amount of P4,550,000. The note provides for 7 equal collections that will start
on December 31, 2018. No interest rate was included in the note. The fair value of the land
and the market value of the note were not determinable. The prevailing rate for a similar
instrument was 8%.
Big Barda Company recorded the exchange at a gain of P1,600,000, the difference between
the face amount of the note and the land’s carrying value of P2,950,000.
14. In relation to the above mentioned information, the net income for 2018 will be
a. Correctly b. Over by c. Over by d. Over by 1,165,859
stated 602,738 895,128
15. The interest income that should be reported in the 2020 comprehensive income statement is
a. 0 b. 240,390 c. 224,005 d. 207,620

Use the following for items 16 - 18


Records of the Dawn Granger Company shows the following relative to product DC
Beginning December 1 3,000 units @ P 30
Purchases December 2,000 units @ P 35 Sal December 3 1,600 units
10 e
December 1,300 units @ P 40 December 8 900 units
12
December 1,200 units @ P 50 December 1,500 units
15 19
December 500 units @ P 60 December 300 units
21 24
December 1,500 units @ P 55 December 1,200 units
30 28
16. Assuming that Dawn Granger Company uses the FIFO periodic system and maintains a 25% mark-up, the cost of goods
sold for December is
a. 180,000 b. 204,500 c. 225,000 d. 259,500
17. Assuming that Dawn Granger Company uses the Average-periodic system and maintains a
20% profit rate, the ending inventory balance is
a. 222,600 b. 194,275 c. 161,895 d. 129,500
18. Assuming that Dawn Granger Company uses the Average-perpetual system and reported sales
of P750,000, the gross profit on sale for December is
a. 483,030 b. 549,970 c. 565,530 d. 632,470

Use the following information for numbers 19 - 20


Godiva Company reports the following data for the month of April
Cost Retail
Inventory, beginning 450,000 1,000,000
Purchases 2,330,000 4,750,000
Purchase returns 114,000 180,000
Purchase discounts 50,500
Transfer in costs 850,000 1,150,000
Transfer out costs 410,500 600,000
Freight-in 95,000
Freight - out 60,000
Mark ups 635,000
Mark up cancelations 355,000
Markdowns 522,500
Markdown cancelations 122,50
0
Sales 4,985,000
Sales returns 750,00
0
Sales discounts 215,000
Employee discounts 775,000
Losses due to shrinkage 265,000
19. Estimated inventory – cost under the average retail method
a. 267,750 b. 275,400 c. 380,625 d. 391,500
20. Cost of sales under the FIFO retail method
a. 2,308,500 b. 2,319,375 c. 2,758,500 d. 2,769,375

Use the following information for numbers 21 - 22


On November 15, 2018, Atlee Company declared a cash dividend of P2.25 per
share on its P9 par value ordinary shares of which 2,000,000 were issued and
outstanding, to holders on record as of December 15 to be distributed on January
15, 2019.
On December 1, 2018 Dolphin Company acquired 300,000 ordinary shares of
Atlee Company at its quoted price of P14.75. Cost incurred in relation to the
acquisition amounted to P132,500. The securities were classified by Dolphin
Company under instruments measured at FV through Profit/Loss.
On December 31, 2018, Atlee Company shares were quoted at P17.50 per share.
Estimated cost to dispose the securities amounted to P115,000
21. The initial measurement of the Investment in Atlee Company is
a. 4,557,500 b. 4,425,000 c. 3,882,500 d. 3,750,000
22. The unrealized gain (loss) reported in the Profit/Loss section of the 2018 comprehensive income statement is
a. 710,000 b. 825,000 c. 1,385,000 d. 1,500,000

Use the following information for numbers 23 - 26


On November 30, 2017, Selena Company acquired 15,000 of Kyle Company’s P8
par value ordinary shares at its quoted price of P9.40 per share as well as 6,000
of Kyle Company’s 9%, P30 par value cumulative, nonparticipating preference
shares at its quoted price of P22. Initial direct cost in acquiring Kyle Company’s
ordinary and preference shares were P5,000 and 4,500 respectively. Kyle
Company has 100,000 ordinary and 12,500 preference shares outstanding. The
securities were designated under instruments measured at FVtOCI.
On December 31, 2017, Kyle Company’s ordinary shares were quoted at P10.30,
while the preference shares were quoted at P23.25. Likewise, estimated disposal
costs were P5,200 and P4,750 respectively. Kyle Company reported a net income
of P580,000 for the year ended December 31, 2017 but did not declare any
dividends.
On December 31, 2018, Kyle Company’s ordinary shares were quoted at P11.65,
while the preference shares were quoted at P24.40. Likewise, estimated disposal
costs were P5,700 and P5,250 respectively. Kyle Company reported a net income
of P720,000 for the year ended December 31, 2018. Total cash dividends
declared amounted to P275,000 on November 20, 2018 for holders on record on
December 20, 2018 to be paid on January 20, 2019.
23. The initial measurement of the investments in Kyle Company’s ordinary shares is
a. 141,000 b. 146,000 c. 132,000 d. 136,500
24. The net unrealized gain/loss included in the equity section for the period ending 2018, in relation to the investment in
Kyle Company’s ordinary shares is
a. 20,250 b. 23,050 c. 28,750 d. 33,750
25. The unrealized gain/loss included in the OCI section for the period ending 2018, in relation to the
investment in Kyle Company preference shares is
a. 6,900 b. 7,400 c. 9,900 d. 10,400
26. The dividend income to be included in the P&L section of the 2018 comprehensive income statement is
a. 31,125 b. 52,400 c. 63,525 d. 77,795
27. On January 1, 2018, Cat Woman Company purchased a new machine on a deferred payment
basis. A down payment of P500,000 was made and 4 annual payments of P150,000 are to be
made beginning on January 1, 2019. The cash price of the machine was P950,000
Due to an employee strike, Cat Woman Company could not install the machine
immediately, and incurred P8,000 of storage costs. Installation and testing were
completed by April 1, 2018. Total installation cost (excluding storage costs)
amounted to P32,000
Cat Woman Company estimates the useful life of the machine to be 8 years with a
residual value of P12,000. Cat Woman Company uses the straight-line method to
record depreciation.

The initial measurement of the machine is


a. 1,340,000 b. 1,332,000 c. 990,000 d. 982,000
28. Halo Company is developing a medicine delivery device. Total expenditures incurred for the
project was P3,450,000 of which P1,152,500 was incurred before 2018, P862,500 from
January 1 to March 31 and P1,435,000 after March 31. Halo Company’s management team
determined that the device will generate probable future economic benefits and has achieved
technical feasibility as of March 31, 2018. Patent registration cost amounted to P45,000. By
July 1, 2018, production and distribution of the medicine delivery device started. Estimated
useful life of the patent for the medicine delivery device is 10 years.
The amortization expense reported in 2018 is

a. 148,000 b. 143,500 c. 74,000 d. 71,750


29. In 2018, Dreamer Company started operations. The pretax income for 2018 was P1,975,000.
Nontaxable income, P115,000; nondeductible expenses, P60,000; future taxable amount,
P550,000; future deductible amount, P360,000; current and expected tax rates, 30%
The taxable income for 2018 is

a. 1,730,000 b. 1,785,000 c. 2,110,000 d. 2,160,000


30. Violet Company adopted a defined benefit plan in reporting its post-retirement benefits. On
January 1, 2018, the following beginning balances were: Projected benefit obligation,
P915,000; Plan assets, P770,000.
Information for 2018 are as follows: Current service cost, P75,000;
Settlement rate 9%; expected rate of return 8%; Actuarial gain on plan assets,
P12,500; Benefits paid, P105,000; Contributions, P60,000
Plan asset balance at December 31, 2018 is
a. 781,800 b. 794,100 c. 799,100 d. 806,800
31. The third millennium ushered in what is commonly described as borderless economic
transactions. Which of the following is the primary factor that accelerated this world
phenomenon?
A. Fast-paced development of Information technology
B. Globalization and e-commerce
C. Uniform international financial reporting standards
D. Uniform international education standards
32. Which of the following statements about Philippine GAAP is (are) false?
I. The Philippine Interpretations Committee issuances are an indication that the IFRSs
are principles-based.
II. The Securities and Exchange Commission (SEC) allows micro entities to use cash basis
of accounting.
III. All reporting enterprises entities in the Philippines including regulated entities are
required to follow full Philippine Financial Reporting Standards (PFRSs) in the
preparation of financial statements
A. I only B. III only C. I and II only D. I, II, and III

33. Which of the following statements on the scope of authority of the PFRSs and the Conceptual
Framework is false?
A. The Philippine Financial Reporting Standards in the Philippines include the PFRSs, the
PASs, and the Philippine Interpretations
B. The Conceptual Framework has the highest level of authority in financial accounting
and reporting practice in the Philippines
C. The Philippine Financial Accounting Standards applies to all reporting enterprises,
whether publicly accountable or small and medium-sized entities
D. The Philippine Interpretations Committee (PIC) assists the FRSC and the public it
serves by addressing newly identified financial reporting or controversial issues in the
Philippines not specifically covered by the PFRSs.

34. Which of the following entities are required to apply the PFRS for SMEs in Philippine
financial reporting?
E. Layette Novelty Store with total total assets of P 300 M and total
liabilities of P 110 M
F. SP Insurance Company with total assets of P 420 M and total liabilities
of P 195 M
G. Rural Bank of Capistrano, Tarlac, with total assets of P 300 M and total
liabilities of P 120 M
H. CJ Foodhouse retails food. It is in the process of issuing securities in a
domestic stock exchange.

35. Which TWO of the following areas of an accountant’ work are not part of financial
accounting?
1) Reporting on parent and subsidiary relationships and transactions
2) Reporting on installment sales and long-term construction contracts
3) Reporting on profitability trends as well as ratios and measurements
4) Reporting on the fairness of presentation of financial position and performance
of an entity in conformity with GAAP.

A. (1) and (2) B. (3) and (4) C. (1) and (3) D. (2) and (3)

36. Which of the following statements about background of accounting is (are) true?
I. The accounting cycle that Luca Pacioli designed is exactly the same as that which the
accounting profession is adopting today.
II. A milestone for the accountancy profession in the current millennium is the uniform
adoption of international accounting standards by all countries of the world.
III. Adoption of the same accounting standards worldwide would have the advantage of
comparability across entities and lower costs in the preparation of financial
information.
Statement I Statement II Statement III
A. False True True
B. False False True
C. True True False
D. True False True

37. International accounting standard-setting employs a “due process system which


A. Is an efficient system for collecting dues from members
B. enables interested parties to express their views on issues under consideration
C. identifies the accounting issues that are the most important
D. requires that all accountants must receive a copy of the financial accounting standards

38. A soundly developed conceptual framework of concepts and objectives should


A. Increase financial statement users' understanding of and confidence in
financial reporting.
B. Enhance comparability among companies' financial statements.
C. Allow new and emerging practical problems to be more quickly solved.
D. All of these.

39. In 2018, INVENTOR Corporation a publicly accountable top-rated manufacturing corporation


incurred research and development costs of P3,000,000 in developing Product YME. On
September 30, management is highly confident that Product YME can be used as a new
raw material for one of its main inventories for sale and has mapped out detailed plans and
target date for its completion no later than 2019.
In its December 31, 2018 statement of financial position, Inventor Corporation should
A. Report Research and Development Expense of P3,000,000 as Product YME is not yet
complete and therefore, there is low probability of benefit flow to the entity
B. Report an Intangible Asset of P750,000 as it has legal ownership of the asset
C. Report an Intangible Asset of P3,000,000 as it has equitable control of the asset
D. Report an Intangible Asset of P750,000 as it has exclusive knowledge and control of
the expected benefit flow even without legal control.

40. Accountable events must meet three criteria for recognition. Which of the following selected
business documents of France Corporation that are presented to you by its management in
2017 will qualify as an accountable event(s)?
A. A purchase commitment signed on December 28, 2016 for P1,000,000 worth of
merchandise inventory to be delivered on April 30, 2017.
B. A purchase order for P 500,000 worth of merchandise
C. A salary voucher for P 350,000 for the last five days of the current year.
D. A invoice for the purchase of machinery, P400,000 under terms 2/10, n/30
E. A newly signed 20-year non-cancellable lease contract for the use of a building with a
useful life of 50 years. The present value of the annual payments for 10 years is P
800,000.
The accountable events that should be recognized in the books of France
Corporation is
A. P400,000 B. P750,000 C P1,500,000 D, P 3,050,000
41. Internal events are
A. not recorded since they happened only within the business
B. not recorded because they are not capable of money measurement
C. recorded because they involve changes in values of the elements of accounting
D. recorded because they involve exchanges of values between the entity and another
party

42. Which of the following accounting measurements may be made with the greatest degree of
objectivity
A. Bad debts expense for the year C. Appraised value of
land at year-end
B. Cost of land owned as at year-end D. Net Income
43. The use of special journal system of recording transactions and events is an application of
which of the following qualities of financial reporting?
A. Faithful representation C. Timeliness
B. Relevance D. Completeness

44. Which of the following statements about financial statements is (are) incorrect?
I. They show the results of the stewardship of management for the resources
entrusted to it by the capital providers.
II. The financial statements are the primary responsibility of both management and
the external auditor after audit.
III. They are prepared at least annually and are directed to the common
information needs of a wide range of statement users.
IV. All economic entities should comply with generally accepted accounting
principles of the accountancy profession for external reporting purposes
A. Statements I and II only C. Statements I, II and IV
B. Statements II and IV only. D. Statements II, III and IV

45. Complete the sentence: Adjusting entries


A. Are often prepared after the statement of financial position date, but dated as
of the statement of financial position date.
B. Are necessary to enable the financial statements to conform to International
Financial Reporting Standard (IFRS) and the qualitative objectives of financial
reporting
C. Include both accruals and deferrals.
D. Are all of (A), (B) and (C) above

46. Which of the following statements about financial accounting is incorrect?


I. General purpose financial statements must be prepared by a certified
public accountant.
II. Financial accounting is a social science that can be influenced by changes
in the legal, political and business environments.
III. Can be stated in any language or dialect of a geographical jurisdiction.
IV. Not all significant information useful to users can be displayed on the face
of the basic financial statements
A. Statement I only C. Statements I and III
only
B. Statements I and II only D. Statements III and IV
only

47. Which of the following types of acquisition of asset and measurement base is (are) property
and logically matched?
Mode of acquisition Measurement base
1. Acquisition of land invested by the owner 1. Fair value
2. Acquisition of equipment by non-monetary exchange 2. Net realizable value
3. Acquisition of asset by long-term credit 3. Equivalent cash price
A. I and 3 only B. 1 and 2 only C. 2 and 3 only D. 1, 2 and 3

48. In which step/s of the accounting cycle is GAAP and the qualitative objectives most applied?
A. Journalizing and adjusting the books C. Journalizing and preparing closing
entries
B. Posting and preparing trial balance D. Preparing the financial statements

49. Which of the following statements about the accounting cycle is true?
A. The first step in posting is to transfer the debit account from the journal to the ledger
B. A periodic adjusting entry affects both a balance sheet and income statement accounts,
or in some rare instances, only income statement accounts.
C. Selecting and analyzing accountable events are part of the recording phase of the
accounting cycle.
D. Adjusting entries are part of the recording phase of accounting
50. The financial information qualities of faithful representation, verifiability, and freedom from
error are typically applied in which of the following steps of the accounting cycle?
A. Journalizing B. Posting C. Trial Balance preparation D.
Adjusting entries

51. Organization was delisted as an asset in the statement of financial position because it fails to
meet which of the following criterion for accountable elements?
A. Probability criterion
B. Measurability criterion
C. Historical event criterion
D. Probability, measurability and historical event criteria

52. The following six adjusting entries were recorded by RNQ Corp. at the end of the fiscal year:
(1) Bank service charge 500 (4) Wages expense 50,000
Cash 500 Wages payable
50,000
(2) Unearned rent 7,400 (5) Advertising expense 6,000
Rent revenue 7,400 Prepaid advertising
6,000
(3) Bad debts expense 18,000 (6) Prepaid rent 12,000
Allowance for bad debts 18,000 Rent expense
12,000
If the firm reverses all adjusting journal entries which should be
appropriately reversed, which of the six adjusting journal entries would be
reversed?
A. (3), (4) and (6)
B. (1), (2) and (5)
C. (4), (6)
D. All six adjustments should be reversed

53. Which of the following statements does not pertain directly to the Going-Concern
assumption of accounting?
A. Assets and liabilities should be classified in the statement of financial position as to
“current” or “non-current”
B. Threats to the ability of an entity to operate as a going concern, such as a troubled-debt
restructuring arrangement should be disclosed in the notes to financial statements
C. Income and expenses should be recognized as these events occur, even if cash is not yet
received or paid.
D. Conceptually, the Accrual assumption is related to the Going Concern assumption

54. Under the revised Conceptual Framework, which of the following are among the enhancing
qualitative objectives of financial accounting?
A. Relevance D. Faithful representation G. Comparability
B. Neutrality E. Verifiability H. Freedom from error
C. Understandability F. Timeliness I. Completeness
A. A, D, and F C. E, G, H
B. C, E, F and G D. D, E and F

55. The International Accounting Standards Board’s conceptual framework includes a cost-
benefit constraint. Which of the following best describes the cost-benefit constraint?
A. The benefits of the information must be greater than the costs of providing it.
B. Financial information should be free from cost to users of the information.
C. Costs of providing financial information are not always evident or measurable,
but must be considered.
D. All of the choices are correct.

56. Which of the following statements pertaining to accounting measurement is (are) true?
1. All monetary assets and liabilities should be measured at fair value
2. Assets acquired in a non-monetary exchange should be measured at
fair value as it does not involve a cost sacrifice
3. Non-monetary items are those whose values are affected by changing
prices.
A. Statement I only C. Statements II and III only
B. Statement III only D. Statements I, II and III

57. Expensing the cost of an inexpensive waste receptacle which has a useful life of 3 years is
A. a violation of the definition of an asset
B. an application of cost / benefit constraint
C. an application of the time period assumption
D. a violation of the expense recognition principle of systematic and rational allocation
58. Which of the following statements about the qualitative characteristics is (are) true?
I. Relevance is the capacity of information to make a difference in
decision-making by helping users form predictions about outcome of
past, present and future events or confirm /correct prior expectations
II. The quality of faithful representation assures readers that the financial
information is free from bias and faithfully represents what it purports
to show, including adequate disclosure of significant information
III. According to the IASB Conceptual Framework, the attainment of the
qualitative objectives of accounting becomes difficult at times
because of cost constraints
A. I and II only C. II and III only
B. I and III only D. I, II and III

59. After closing the books but before the financial statements are authorized for issue,
bookkeeper discovered an omitted invoice of the company for the purchase of staplers and
punchers amounting to P250. Since the total property and equipment of the entity amount to
P 150 Million, he did not make an entry to record the purchase as he believes that the amount
involved is not material anyway. This is
A. an application of the materiality rule
B. a violation of the principle of recognition and measurement of an element of
accounting
C. justifiable by the cost/benefit constraint concept in the recognition of accounting
elements
D. a violation of the accounting entity assumption

60. Which of the following statements about the concept of measurement or valuation in
Accounting is (are) True?
I. Under current GAAP, as a general rule, the primary basis of measurement of
assets upon acquisition is historical cost.
II. There are some instances when assets are initially measured on the basis
of fair value
III. The final valuation of assets and liabilities in the balance sheet are a
mixture of costs and values
IV. According to IFRS, Under no circumstances is price-level accounting
acceptable as an alternative measurement in accounting in present-day
GAAP
A. I and II only C. I, II, and III only
B. I, II and IV only D. I, II, III and IV

END OF EXAMINATION

You might also like